24 olimpiada iberoamericana de matemáticas

77
24 Olimpiada Iberoamericana de Matem´ aticas Septiembre 17-27 2009 Quer´ etaro, M´ exico

Upload: lyquynh

Post on 02-Jan-2017

293 views

Category:

Documents


7 download

TRANSCRIPT

Page 1: 24 Olimpiada Iberoamericana de Matemáticas

24 Olimpiada Iberoamericana

de Matematicas

Septiembre 17-27 2009Queretaro, Mexico

Page 2: 24 Olimpiada Iberoamericana de Matemáticas

Sociedad Matematica MexicanaOlimpiada Mexicana de Matematicas

Page 3: 24 Olimpiada Iberoamericana de Matemáticas

Indice

Invitacion a la 24 Olimpiada Iberoamericana de Matematicas 1

Patrocinadores 3

Ediciones anteriores Olimpiada Iberoamericana de Matematicas 4

Reglamento Permanente de la Olimpiada Iberoamericana de Matematicas 5

Reglamento de la Copa Puerto Rico al paıs de mejor progreso en cadaOlimpiada Iberoamericana de Matematicas 11

Comites OIM 14

Informacion para visitantes 18

Mexico 18

Queretaro 18

Queretaro (capital) 18

San Miguel de Allende 19

Sitios de visita oficial 20

Guanajuato 20

CIMAT 20

Bernal 21

Tequisquiapan 21

Programa 24 OIM 24

Ceremonia de inauguracion 24 OIM 27

Carta de la reunion del CEA 29

Participantes 30

Problemas y Soluciones de la 24 OIM 36

iiiiiiiii

Page 4: 24 Olimpiada Iberoamericana de Matemáticas

iv INDICE

Actas de las reuniones de jurado de la 24 OIM 46

Premios de la 24 OIM 56

Puntaje por participante 58

Distribucion por problema y puntaje 63

Promedio por problema 65

Medallas 66

Medallas por paıs 69

Resultado Copa Puerto Rico 70

Ceremonia de clausura 24 OIM 71

24 Olimpiada Iberoamericana de Matematicas, Mexico.

Page 5: 24 Olimpiada Iberoamericana de Matemáticas

Invitacion a la 24 OlimpiadaIberoamericana de Matematicas

Mexico, D. F., a 25 de mayo de 2009.

El Comite Organizador de la 24 Olimpiada Iberoamericana de Matematicas (OIM) se com-place en invitar a su paıs a participar en la vigesima cuarta edicion de la Olimpiada Ibe-roamericana de Matematicas que se realizara en la ciudad de Queretaro, Queretaro, Mexico,entre los dıas 17 y 27 de septiembre de 2009. Este evento esta organizado por la So-ciedad Matematica Mexicana y patrocinado fundamentalmente por la Secretarıa de Edu-cacion Publica y el Consejo Nacional de Ciencia y Tecnologıa.

La delegacion que representara a su paıs en la 24 OIM debera estar conformada de la siguientemanera:

• Un equipo de hasta 4 estudiantes. Ninguno de los integrantes de las delegacionesdebera haber participado en dos Olimpiadas. Asimismo, los integrantes deberan tenermenos de 18 anos cumplidos al 31 de diciembre de 2008.

• Un profesor que representara al paıs participante en calidad de Jefe de Delegacion.

• Un profesor que acompanara a los estudiantes representando al paıs en calidad deTutor.

Como se especifica en el Reglamento General de la Olimpiada Iberoamericana de Matema-ticas, el Comite Organizador de la 24 OIM cubrira todos los gastos de alojamiento y ali-mentacion de las delegaciones de los diferentes paıses iberoamericanos. Los gastos de trans-porte aereo o terrestre, desde y hasta, la ciudad de Queretaro seran responsabilidad del paısparticipante, ası como cualquier gasto adicional no estipulado por la organizacion del evento.

Asimismo, le informamos que los dıas 17 y 18 de septiembre del mismo ano sera realizadoel V Simposio Iberoamericano de Educacion Matematica con enfasis en la solucion de pro-blemas. La delegacion representante en la 24 OIM tambien esta cordialmente invitada aparticipar en el V Simposio. Durante el Simposio, el Comite Organizador cubrira los gastos

111

Page 6: 24 Olimpiada Iberoamericana de Matemáticas

2 Invitacion a la 24 Olimpiada Iberoamericana de Matematicas

de alojamiento y alimentacion para un profesor representante y los gastos de otros partici-pantes deberan ser cubiertos por su organizacion.

En la pagina http://www.oim2009.org se encuentra toda la informacion del evento, en par-ticular los formularios de inscripcion de los equipos que cada delegacion debe llenar.

Favor de confirmar su participacion, sea por correo electronico o por fax, a mas tardar el dıa30 de junio de 2009.

Esperando tener el placer y la honra de contar con la presencia de su paıs en la 24 OlimpiadaIberoamericana de Matematicas, le enviamos nuestro mas cordial saludo.

Dra. Radmila Bulajich ManfrinoPresidenta del Comite Organizador

Mtro. Alonso LujambioSecretario de Educacion Publica

24 Olimpiada Iberoamericana de Matematicas, Mexico.

Page 7: 24 Olimpiada Iberoamericana de Matemáticas

Patrocinadores

Secretarıa de Educacion PublicaUniversidad Autonoma del Estado de MorelosConsejo Nacional de Ciencia y TecnologıaCentro de Investigacion en Matematicas A.C.Facultad de Ingenierıa de la Universidad Autonoma de QueretaroFacultad de Ciencias de la UNAMConsejo de Ciencia y Tecnologıa del Estado de QueretaroUniversidad de GuanajuatoSecretarıa de Educacion Publica del Estado de QueretaroSecretarıa de Turismo del Estado de Queretaro

333

Page 8: 24 Olimpiada Iberoamericana de Matemáticas

Ediciones anteriores OlimpiadaIberoamericana de Matematicas

1985 - Villa de Leyva, Colombia1987 - Paysandu, Uruguay1988 - Lima, Peru1989 - La Habana, Cuba1990 - Valladolid, Espana1991 - Cordoba, Argentina1992 - Caracas, Venezuela1993 - Mexico D. F., Mexico1994 - Fortaleza - Ceara, Brasil1995 - Region V, Chile1996 - San Jose, Costa Rica1997 - Guadalajara - Jalisco, Mexico1998 - Puerto Plata, Republica Dominicana1999 - La Habana, Cuba2000 - Caracas, Venezuela2001 - Minas, Uruguay2002 - San Salvador, El Salvador2003 - Mar del Plata, Argentina2004 - Castellon, Espana2005 - Cartagena, Colombia2006 - Guayaquil, Ecuador2007 - Coimbra, Portugal2008 - Salvador - Bahıa, Brasil

444

Page 9: 24 Olimpiada Iberoamericana de Matemáticas

Reglamento Permanente de laOlimpiada Iberoamericana deMatematicas

1. ASPECTOS GENERALES

1.1 La Olimpiada Iberoamericana de Matematicas, OIM, es una competicion de Matema-ticas dirigida a estudiantes que no hayan cumplido 18 anos de edad al 31 de diciembre delano inmediato anterior a la celebracion de la Olimpiada, se organiza anualmente y participanlos paıses iberoamericanos.

1.2 Las Olimpiadas Iberoamericanas de Matematica (O.I.M.), se efectuaran en el paısseleccionado con el auspicio y apoyo de los organismos (nacionales e internacionales), enti-dades o personas que deseen colaborar con estos eventos y esten debidamente autorizadospor el Comite Organizador de la Olimpiada particular de la que se trate.

1.3 Sera responsabilidad del paıs sede la designacion del Comite Organizador.

1.4 El Comite Organizador de cada Olimpiada determinara el perıodo durante el cual serealizara el evento en cuestion, tratando de ajustarse a los intereses generales de los paısesparticipantes y a la tradicion.

2. OBJETIVOS DE LA OLIMPIADA

La O.I.M. es una competicion entre jovenes estudiantes de paıses iberoamericanos, cuyosobjetivos son:

• Estimular el estudio de las matematicas en los paıses de la region.

• Descubrir y estimular a los jovenes con talento para el estudio de las matematicas enla region.

• Auspiciar relaciones de amistad entre estudiantes, profesores y maestros de matematicasde los paıses participantes.

555

Page 10: 24 Olimpiada Iberoamericana de Matemáticas

6 Reglamento Olimpiada Iberoamericana de Matematicas

• Crear una oportunidad para el intercambio de experiencias educativas entre los profe-sores de matematicas de la region.

3. PARTICIPACION

3.1 El paıs sede debera invitar a los siguientes paıses iberoamericanos: Argentina, Bolivia,Brasil, Chile, Colombia, Costa Rica, Cuba, Ecuador, El Salvador, Espana, Guatemala, Hon-duras, Mexico, Mozambique, Nicaragua, Panama, Paraguay, Peru, Portugal, Puerto Rico,Republica Dominicana, Uruguay y Venezuela. El paıs organizador podra invitar a cualquierotro paıs de lengua espanola o portuguesa.

3.2 Cada paıs invitado tiene derecho a estar representado por un equipo de hasta cuatroestudiantes, un profesor Jefe de Delegacion y un Tutor de los alumnos.

3.3 Los estudiantes concursantes deberan satisfacer los requerimientos siguientes:

a) No haber cumplido los 18 anos de edad al 31 de diciembre del ano inmediato anteriora la celebracion de la Olimpiada.

b) No haber participado en dos Olimpiadas Iberoamericanas anteriores.

3.4 El Jefe de Delegacion formara parte del Jurado de la Olimpiada y residira con losmiembros de este Jurado, sin poder establecer contacto con los estudiantes concursantes nicon los profesores tutores hasta que hayan finalizado las pruebas de la Olimpiada.

3.5 El Tutor se regira por el artıculo anterior a partir del momento en que, por algunmotivo justificado, haya debido integrar el Jurado Internacional o establecido contacto conalgun miembro del mismo.

4. FINANCIAMIENTO DE LA OLIMPIADA

4.1 Los organizadores cubriran los gastos normales de estancia, alojamiento, alimentacion,y traslados internos de las delegaciones, durante el perıodo de las Olimpiadas. Queda enten-dido que los traslados incluyen la recepcion y despedida de las delegaciones desde y hasta elaeropuerto.

4.2 Los familiares y observadores acreditados podran acompanar a las delegaciones, peroestaran sujetos a las reglas de financiamiento y organizacion que determine el Comite Orga-nizador del evento.

4.3 El pasaje internacional de los participantes es responsabilidad de cada paıs invitado.No obstante, el Comite Organizador tratara de buscar alternativas que faciliten la finan-ciacion de los pasajes.

24 Olimpiada Iberoamericana de Matematicas, Mexico.

Page 11: 24 Olimpiada Iberoamericana de Matemáticas

Reglamento Olimpiada Iberoamericana de Matematicas 7

4.4 Para colaborar con la organizacion de la Olimpiada siguiente, el paıs sede cubrira losgastos normales de estancia de dos observadores del paıs que sera la sede al ano siguiente.

4.5 Cada paıs invitado debera garantizar que su delegacion viaja con un seguro que cubracualquier eventualidad medica o accidente que ocurra a cada uno de sus miembros.

5. DE LOS PROBLEMAS MATEMATICOS PARA LA COMPETICION

5.1 Cada paıs invitado podra proponer hasta seis problemas al Comite Organizador. Lafecha lımite de entrega de estos problemas sera fijada por ese Comite.

5.2 El Comite Organizador de la Olimpiada tendra una reserva de 18 problemas.

5.3 Los problemas versaran sobre los distintos campos de la Matematica de la EducacionPreuniversitaria y seran variados en cuanto al nivel de dificultad y a los temas que se selec-cionen.

6. LAS PRUEBAS

6.1 La competicion constara de dos pruebas escritas de cuatro horas y media de duracioncada una, que se realizaran en dos dıas consecutivos. Cada prueba constara de tres proble-mas.

6.2 Cada concursante trabajara individualmente. Las respuestas y soluciones se consigna-ran en papel previsto por el Comite Organizador.

Los unicos instrumentos cuyo uso se permitira seran los necesarios para escribir y dibujar.No esta permitido el uso de libros, libretas de notas, tablas y calculadoras.

6.3 Durante los primeros treinta minutos de cada dıa, a partir del comienzo de las prue-bas, los concursantes podran formular preguntas por escrito al Jurado. Estas preguntasversaran sobre posibles aclaraciones de los enunciados de los problemas.

7. EL JURADO DE LA OLIMPIADA IBEROAMERICANA

7.1 El desarrollo de la Olimpiada es responsabilidad del Jurado, que estara integrado porlos Jefes de Delegaciones participantes y por un miembro designado por el Comite Organi-zador, quien lo presidira. El Jurado Internacional es la maxima autoridad de la Olimpiadadurante el desarrollo de la misma y sus decisiones seran inapelables. Entre una Olimpiaday la siguiente la autoridad recaera sobre el Comite Ejecutivo-Asesor de la Olimpiada y elComite Organizador de la Olimpiada siguiente.

24 Olimpiada Iberoamericana de Matematicas, Mexico.

Page 12: 24 Olimpiada Iberoamericana de Matemáticas

8 Reglamento Olimpiada Iberoamericana de Matematicas

7.2 El Comite Organizador designara un Equipo de Coordinacion con primero y segundojefes. La funcion del equipo es la de coadyuvar a una evaluacion justa y uniforme de laspruebas.

7.3 A las reuniones del Jurado, asistiran como asesores de la presidencia, dos vicepresi-dentes y un secretario designados por el Comite Organizador, ası como los jefes del Equipode Coordinacion.

7.4 Salvo los casos que hubieran sido acordados previamente por el Comite Organizador,los observadores, coordinadores y cualquier otro personal, deberan contar con la autorizaciondel presidente para asistir a las reuniones del Jurado. Los asistentes autorizados podran tenervoz, pero no derecho a voto.

7.5 Una vez iniciada la prueba el segundo dıa, el profesor Tutor se incorporara a lasdeliberaciones del Jurado.

7.6 En las reuniones del Jurado cada Jefe de Delegacion o la persona que eventualmentelo represente en las votaciones tendra un voto. En caso de empate se emitira el voto delPresidente del Jurado, que sera dirimente.

7.7 El Jurado podra formar subcomites con el fin de asesorar sobre aquellas cuestionesespecıficas para las cuales hayan sido designados.

7.8 El Jurado debera:

a) Seleccionar de entre los problemas propuestos, los seis que conformaran las Pruebas dela Olimpiada.

b) Decidir sobre la formulacion precisa de los enunciados de los problemas en espanol yen portugues.

c) Decidir sobre posibles respuestas a las preguntas que los participantes formulen durantelos primeros treinta minutos de cada prueba.

d) Tomar decisiones en caso de que se presente diferencia de opiniones entre el Equipo deCoordinacion y el Jefe de alguna Delegacion.

e) Decidir sobre la distribucion de premios (Medallas, Copa Puerto Rico, MencionesHonorıficas y Premios Especiales)

8. CALIFICACION DE LAS PRUEBAS

8.1 El Comite Organizador designara, dentro del Equipo de Coordinacion, un Tribunalde Coordinacion para cada problema propuesto, uno de cuyos miembros figurara como res-ponsable. Cada uno de estos tribunales establecera las pautas para la calificacion de los

24 Olimpiada Iberoamericana de Matematicas, Mexico.

Page 13: 24 Olimpiada Iberoamericana de Matemáticas

Reglamento Olimpiada Iberoamericana de Matematicas 9

problemas y las presentara al Jurado para su conocimiento, previo a la calificacion de laspruebas.

8.2 Las respuestas de los estudiantes llegaran a los Jefes de Delegacion a traves del ComiteOrganizador, quien guardara fotocopias de todos los examenes con el fin de garantizar nor-mas de seguridad para el mejor desenvolvimiento del evento. Las respuestas dadas por unestudiante seran calificadas separadamente por el Jefe de Delegacion de su paıs, auxiliadopor el Tutor, y por el Tribunal de Coordinacion. Para esta calificacion se seguiran unaspautas previamente establecidas por los Tribunales de Coordinacion y presentadas para suconocimiento al Jurado Internacional.

8.3 En la calificacion de las pruebas el texto presentado por los estudiantes debera serpreservado de cualquier alteracion. Todo senalamiento de los profesores debera quedar clara-mente diferenciado, ya que en caso contrario se anulara la pregunta.

8.4 El Jefe de Delegacion y el Tutor presentaran ante el Tribunal de cada problema unaevaluacion fundamentada de la solucion de cada estudiante de su equipo. El Tribunal delproblema en cuestion debera establecer la calificacion correspondiente. Si el Jefe de Dele-gacion y el Tribunal no pueden llegar a un acuerdo sobre la calificacion de un problema enparticular, se dara intervencion al Jefe del Equipo de Coordinacion. En caso de mantenerseel desacuerdo, este sera llevado ante el Jurado de la Olimpiada, el que decidira sobre lacalificacion.

8.5 La calificacion acordada entre los responsables de cada paıs y el Tribunal de Coordi-nacion se hara constar en una carpeta con las pruebas del concursante, y en un documentoadicional con las firmas del Jefe de Delegacion del paıs en cuestion y el Jefe del Tribunal delproblema de que se trate.

8.6 En las sesiones de coordinacion correspondientes a los integrantes de la delegaciondel paıs organizador, debera estar presente en calidad de observador un Jefe o Tutor deDelegacion de alguno de los paıses participantes distinto del paıs sede.

9. PREMIOS Y DIPLOMAS

9.1 Se otorgaran medallas de oro, plata y bronce, ademas se otorgaran menciones honorıfi-cas a los estudiantes que hayan resuelto un problema perfecto, pero no obtengan la pun-tuacion mınima necesaria para ganar una medalla. Asimismo, se podra otorgar uno o maspremios especiales a aquellas soluciones que los miembros del Equipo de Coordinacion con-sideren muy significativas u originales. El Jefe de los Tribunales de Coordinacion sera lapersona encargada de presentar al Jurado Internacional, cualquier propuesta de premio es-pecial.

24 Olimpiada Iberoamericana de Matematicas, Mexico.

Page 14: 24 Olimpiada Iberoamericana de Matemáticas

10 Reglamento Olimpiada Iberoamericana de Matematicas

9.2 El numero total de medallas de oro, plata y bronce sera aproximadamente el 50% deltotal de participantes. La razon entre las cantidades de medallas de oro, plata y bronce quese otorguen sera aproximadamente de 1:2:3.

9.3 Cada participante recibira un diploma acreditativo de su participacion.

9.4 Las medallas y diplomas seran entregados en un Acto de Clausura.

10. DE LAS PROXIMAS OLIMPIADAS

En cada Olimpiada el Jurado Internacional seleccionara de entre las peticiones de sederecibidas, la sede de la Olimpiada del tercer ano, a partir de esa. En caso de empate delos votos de los Jefes de Delegaciones, el Presidente del Jurado emitira un voto dirimente.

11. COMITE EJECUTIVO-ASESOR. CEAOIM

Con el fin de garantizar la continuidad en los criterios de organizacion de la Olimpiada yatender a las posibles contingencias que pudieran surgir, entre cada Olimpiada existira unComite Ejecutivo-Asesor, CEAOIM, compuesto por cinco personas: un representante desig-nado por cada uno de los dos paıses que han sido la sede en las dos Olimpiadas anteriores,un representante por cada una de de las dos proximas Olimpiadas y un representante delJurado Internacional. Este Comite actuara entre la ultima reunion del Jurado Internacionalde una Olimpiada y la primera reunion del Jurado de la siguiente Olimpiada. El CEAOIMse regira por un reglamento que se elaborara para tal fin y que debera ser aprobado por elJurado Internacional.

El representante del Jurado Internacional durara un ano en sus funciones y sera de caracterrotativo, siguiendo el orden alfabetico de los paıses asistentes a la Olimpiada. El primerrepresentante del Jurado Internacional se designara por sorteo y luego continuara el ordenlexicografico. Si el paıs que le corresponde en un ano, ya esta representado en el Comite, ono asistio a esa Olimpiada, entonces le toca el turno al siguiente paıs en el orden establecidoy que no este representado en el Comite.

12. OTROS ASUNTOS

Cualquier reglamentacion contradictoria, situacion no recogida en este Reglamento, asuntoespecial o modificacion de reglas, sera decidida por el Jurado Internacional, con el votodirimente del Presidente del Jurado.

Quedan derogados todos los acuerdos tomados previamente a la sancion de este Reglamen-to o que no figuren en el.

Coimbra, 13 de Septiembre de 2007.

24 Olimpiada Iberoamericana de Matematicas, Mexico.

Page 15: 24 Olimpiada Iberoamericana de Matemáticas

Reglamento de la Copa Puerto Ricoal paıs de mejor progreso en cadaOlimpiada Iberoamericana deMatematicas

El Jurado Internacional de la XIV Olimpiada Iberoamericana de Matematicas, reunido enLa Habana, Cuba, el 11 de septiembre de 1999, estudiada la propuesta de cambio presentadapor el Jefe de Delegacion de Mexico, acuerda modificar el Reglamento de la Copa PuertoRico, que queda establecido del siguiente modo:

ANTECEDENTES

La delegacion de Puerto Rico, en el marco de la IV Olimpiada Iberoamericana deMatematicas (O.I.M.), propuso otorgar cada ano un trofeo al paıs de mejor progreso relativoen las O.I.M., con el objetivo de estimular el desarrollo de los equipos, independientementedel nivel absoluto alcanzado por cada paıs. Al proponer este estımulo, se comprometio a quesu paıs entregarıa cada ano el referido trofeo bajo la denominacion de Copa Puerto Rico. Laproposicion fue aceptada por unanimidad por el Jurado Internacional de la IV O.I.M. Final-mente en Medina del Campo (Valladolid, Espana) fue aprobado, tambien por unanimidad elReglamento propuesto por la Delegacion Cubana, siendo modificado en San Jose de CostaRica.

REGLAMENTO

- Art. 1 Para que un paıs pueda optar por la Copa Puerto Rico en la O.I.M. del anox + 2, debe satisfacer los requisitos siguientes:

(a) Haber participado en las olimpiadas de los anos x y x + 1.

(b) participar con equipo completo.

- Art. 2 El salto de progreso relativo S, alcanzado por un paıs en la O.I.M. del ano x+2,y que satisface los requerimientos del Artıculo 1, es un numero que se determinara medianteel procedimiento siguiente:

111111

Page 16: 24 Olimpiada Iberoamericana de Matemáticas

12 Reglamento Copa Puerto Rico

(a) Se sumaran los puntos obtenidos por ese paıs durante las olimpiadas de los anos xy x + 1, y se dividira esa cantidad por el total de alumnos que integraron los equipos deesos dos anos. Este numero se multiplicara por 100 y se dividira entre el promedio totalde calificaciones de los anos x y x + 1 (esto es, la suma de todas las calificaciones de todoslos alumnos participantes en las olimpiadas x y x + 1, dividida entre el numero total departicipantes en esas olimpiadas).

Este numero representa el promedio normalizado P que obtuvo cada paıs en los dosultimos anos.

(b) El total de puntos alcanzado por el paıs en la O.I.M del ano x + 2 se divide entre 4y entre el promedio general de calificaciones de ese ano, y se multiplica por 100. El numeroobtenido, Q, es el promedio normalizado por paıs en esta Olimpiada.

(c) El salto se define mediante la formula:

S = Q − (11/10)P

- Art. 3 El paıs ganador de la Copa Puerto Rico es aquel que presenta el mayor salto deprogreso relativo S.

En caso de empate entre dos o mas paıses, se seguira el esquema ordenado siguiente,relativo a la Olimpiada x + 2, por eliminaciones sucesivas, hasta obtener el paıs ganador.

(a) El paıs que presenta mayor Q.(b) El paıs ganador de mas medallas de oro.(c) El paıs ganador de mas medallas de plata.(d) El paıs ganador de mas medallas de bronce.(e) El paıs ganador de mas menciones.

Art. 4 En caso de presentarse cualquier situacion no contemplada en este Reglamento,el Jurado Internacional tomara decisiones a partir de las proposiciones de sus integrantes,las cuales se tomaran por mayorıa de votos, una por cada paıs, y con el voto dirimente delPresidente del Jurado en caso de empate en la votacion.

JUSTIFICACIONES

El progreso no puede medirse sin participacion previa. Por otra parte, un paıs con de-sarrollo puede decaer en la olimpiada del ano x + 1 y recuperar su forma tradicional en laolimpiada del ano x + 2, con lo cual podrıa ganar la Copa sin mucho merecerlo. Por talmotivo se propone considerar la actuacion de los dos anos anteriores.

24 Olimpiada Iberoamericana de Matematicas, Mexico.

Page 17: 24 Olimpiada Iberoamericana de Matemáticas

Reglamento Copa Puerto Rico 13

El progreso debe medirse por puntos y no por medallas, para dar preferencia al trabajocolectivo del equipo y no a la presencia eventual de un joven muy talentoso, que no repre-sentarıa el verdadero desarrollo del paıs.

En los promedios de los anos x y x+1, se considera promedio por alumno para no afectara un paıs que en uno de esos anos, por razones economicas o imprevistas, no puede concursarcon equipo completo.

Para ganar la Copa, sin embargo, se necesita concurrir con equipo completo debido a queen otro caso, podrıa concurrir con solo uno o dos alumnos brillantes y obtener un promediomuy alto por alumno. Ademas, parece mas justo que se le otorgue la Copa a un paıs conequipo completo.

La nueva formula para calcular S proviene de reemplazar el coeficiente 9/10, que se colocopara compensar en algo a los paıses de mas desarrollo relativo, por el de 11/10 que favorecea los paıses con menor desarrollo relativo en las OIM.

El reglamento sigue obedeciendo al espıritu en el cual se aprobo, es decir, progreso de unano a otro.

Los problemas propuestos en una Olimpiada determinada pueden ser mas difıciles queen las dos anteriores. Al dividir entre los promedios generales de calificaciones en los anoscorrespondientes se elimina la posibilidad de que paıses cuyas bajas calificaciones no depen-den del nivel de la prueba (por ejemplo una constante de 0) esten por encima de otros paısesque, a pesar de haber trabajado relativamente mejor que en anos anteriores, obtengan pun-tuacion negativa a causa de la mayor dificultad del examen. El factor 100 tiene el propositode considerar numeros de valor absoluto entre 0 y 100.

24 Olimpiada Iberoamericana de Matematicas, Mexico.

Page 18: 24 Olimpiada Iberoamericana de Matemáticas

Comites OIM

Comite AcademicoFlorian Luca (Comite de Seleccion de Problemas)Jose Antonio Gomez Ortega (Presidente del Jurado)Rogelio Valdez Delgado (Jefe de Coordinadores)Radmila Bulajich Manfrino (Vicepresidente del Jurado)

Comite Ejecutivo Asesor de la OIM 2008-2009Mexico Radmila Bulajich Manfrino (Presidenta)Brasil Luzinalva Miranda de Amorin (Secretaria)Paraguay Gabriela Gomez Pascuali (Vocal)Portugal Joana Teles Correira (Vocal)Espana Juan Manuel Conde (Vocal)

Comite Ejecutivo Asesor de la OIM 2009-2010Paraguay Gabriela Gomez Pascuali (Presidenta)Mexico Radmila Bulajich Manfrino (Secretaria)Costa Rica Mario Marın Sanchez (Vocal)Guatemala Mayra Castillo (Vocal)Brasil Carlos Yuzo Shine (Vocal)

Comite FinancieroRadmila Bulajich ManfrinoCarmen Sosa GarzaIgnacio Barradas Bribiesca

Comite de Seleccion de ProblemasPatricio Tlacaelel Alva PuffleauDavid Cossio RuizJose Antonio Gomez OrtegaManuel Angel Guevara LopezIvan Joshua Hernandez MaynezFlorian Luca (Jefe del Comite)Leonardo Ignacio Martınez SandovalJesus Efren Perez TerrazasPablo Soberon Bravo

141414

Page 19: 24 Olimpiada Iberoamericana de Matemáticas

Comites OIM 15

Enrique Trevino LopezRogelio Valdez DelgadoEduardo Velasco BarrerasCarlos Villalvazo JaureguiHugo Villanueva Mendez

Comite de CoordinadoresRogelio Valdez Delgado (Jefe de Coordinadores)Bernardo Abrego LermaPatricio Tlacaelel Alba PufleauJesus Rodrıguez VioratoCarlos Villalvazo JaureguiEdgardo Roldan PensadoJoshua Hernandez MaynezManuel Angel Guevara LopezEduardo Velasco BarrerasHugo Villanueva MendezDavid Cossio RuizHumberto Montalvan GamezAna Rechtman BulajichJesus Efren Perez TerrazasAntonio Olivas MartınezEnrique Trevino LopezJulio Rodrıguez HernandezMarıa Eugenia Guzman FloresCarlos Jacob Rubio Barrios

Comite de Desarrollo del EventoAnne Alberro Semerena (Coordinadora)Carmen Sosa Garza

Comite de Relaciones PublicasKelly Scoggins Flower (Coordinadora)

Comite de HotelesKelly Scoggins Flower (Coordinadora en Queretaro)Anne Alberro Semerena (Coordinadora en San Miguel de Allende)

Comite de SimposioCarmen Sosa Garza

24 Olimpiada Iberoamericana de Matematicas, Mexico.

Page 20: 24 Olimpiada Iberoamericana de Matemáticas

16 Comites OIM

Comite de EventosCarmen Sosa GarzaPatricia Spındola YanezCarlos Pescador Spındola

Comite de TransporteCarlos Pescador Spındola (Coordinador en Queretaro)Ivan Gonzalez GarcıaVictor Antonio Aguilar ArteagaAnne Alberro Semerena (Coordinadora en San Miguel de Allende)

Comite de ComputoLuis Cruz Romo (Coordinador)Jose Antonio Climent HernandezRafael E. Porras Trejo

Comite de AlumnosIgnacio Barradas Bribiesca (Coordinador)

Comite de ExamenOlga Rivera Bobadilla (Coordinadora)Rita Vazquez Padilla

Comite EditorialMontserrat Garcıa Campos (Coordinadora)Luis Cruz RomoRadmila Bulajich Manfrino

Comite de PromocionalesRadmila Bulajich ManfrinoOlga Rivera BobadillaCarmen Sosa GarzaAnne Alberro Semerana

Identidad Visual y Diseno GraficoYosune Chamizo Alberro

FotografıaHozcami Arellano Vargas

Pagina webRodrigo Chamizo

24 Olimpiada Iberoamericana de Matematicas, Mexico.

Page 21: 24 Olimpiada Iberoamericana de Matemáticas

Comites OIM 17

Jose Antonio Climent HernandezLesvia Elena Ruiz Velazquez

Comite de ApoyoLucina Parra AguilarTeresa de Jesus Valerio Lopez

GuıasGustavo Adolfo Ramos Palacios ArgentinaEmilio Angulo Perkins BoliviaAlejandro Luis Gonzalez Tokman BrasilMariano Raul Gonzalez Tokman ChileOrlando Ochoa Castillo ColombiaEdgar Gonzalez Arreola Costa RicaJuan Antonio Villeda Resendiz CubaJose Trinidad Barajas Vega EcuadorAntonio de Jesus Torres Hernandez El SalvadorIvonne del Carmen Colorado Chairez EspanaJosue Hernandez Carlos GuatemalaPablo Luis Pena Dıaz Honduras

Julio Cesar Iniguez Alonso Mexico

Teresa de Jesus Iniguez Alonso NicaraguaHector Daniel Banos Cervantes PanamaAbraham Solis Garcıa Ina ParaguayLuis Alberto Barradas Chacon Peru

Laura Cecilia Avila Jauregui PortugalPaulina Tello Mota Puerto RicoGilberto Sanchez de la Isla UruguayEdwin Alain Lopez Romo Venezuela

24 Olimpiada Iberoamericana de Matematicas, Mexico.

Page 22: 24 Olimpiada Iberoamericana de Matemáticas

Informacion para visitantes

Mexico

Mexico es una republica federal integrada por 32 entidades federativas que ocupa la partemeridional de America del Norte. De acuerdo con la Constitucion Mexicana vigente, elnombre oficial del paıs es Estados Unidos Mexicanos y la sede de los poderes de la federaciones la Ciudad de Mexico, cuyo territorio ha sido designado como Distrito Federal.

El territorio mexicano limita al norte con Estados Unidos; al este, con el Golfo de Mexicoy el Mar Caribe; al sureste, con Belice y Guatemala; y al oeste con el Oceano Pacıfico. Lasuperficie mexicana ocupa una extension cercana a los 2 millones de km2, que lo colocan, porsu extension, en la decimocuarta posicion entre los paıses mas grandes del mundo. En esteterritorio, habitan mas de 103 millones de personas; ası, se trata de la nacion hispanohablantecon mayor poblacion, aunque el espanol convive en Mexico con numerosas lenguas indıgenasque son reconocidas como nacionales por el Estado Mexicano.

Queretaro

El Estado de Queretaro ha sido beneficiado por su ubicacion geografica: en varios puntos delestado se encuentran importantes casonas coloniales, producto, en parte, de la evangelizaciony de rutas como el Camino Real de Tierra Adentro.

Hoy, Queretaro es reconocido como un estado dinamico en el que el buen vivir se conjugacon la tranquilidad y belleza de sus pueblos y ciudades. Distinguido con dos PatrimoniosMundiales (el Centro Historico de Santiago de Queretaro y las misiones franciscanas), unPueblo Magico (Bernal) y una Reserva de la Biosfera (la Sierra Gorda), Queretaro es unestado que tiene, de lo bueno, todo.

Queretaro (capital)

Queretaro es uno de los 18 municipios del estado mexicano de Queretaro. La cabeceramunicipal es la Ciudad de Queretaro, tambien capital del estado.

El municipio de Queretaro, con 734,139 habitantes en el 2005 (segun el INEGI), es latercera mas poblada de la region del Bajıo, despues de Leon y Morelia, aunque su zonametropolitana (918,100 hab) supera a la de esta ultima ciudad. A nivel nacional, es la

181818

Page 23: 24 Olimpiada Iberoamericana de Matemáticas

San Miguel de Allende 19

undecima zona metropolitana mas poblada. Su tasa de crecimiento anual fue de 2.7% en elperıodo 2000 - 2005.

La historica y colonial ciudad de Santiago de Queretaro tiene el orgullo de haber sidodesignada como Patrimonio de la Humanidad por la UNESCO. Su cercanıa con la Ciudadde Mexico de tan solo 222 km, y su ubicacion en la region central del Paıs la hacen unade las ciudades mas importantes de Mexico. Su superficie es de 11,449 km2, se encuentra a1,800 m sobre el nivel del mar y tiene una poblacion aproximada de 1,200,000 habitantes.

San Miguel de Allende

San Miguel de Allende es una ciudad del estado mexicano de Guanajuato. Tiene una alturade 1,910 m sobre el nivel del mar y esta situada a 274 km, de la Ciudad de Mexico y a 97km de Guanajuato. Es considerado un “pueblo magico”.

El 7 de julio de 2008, fue inscrita por la Unesco en el Patrimonio cultural de la Hu-manidad. Bajo el tıtulo de Ciudad fortificada de San Miguel y santuario de Jesus Nazarenode Atotonilco, la distincion se otorgo debido a su aporte cultural y arquitectonico al Barrocomexicano y a su importancia en la lucha de Independencia de Mexico de Espana.

El pueblo se destaco prominentemente durante la Guerra de Independencia de Mexico.El General Ignacio Allende, nativo de San Miguel, fue un lıder clave en la guerra de inde-pendencia en contra de Espana. A Allende, quien fue capturado saliendo del paıs en buscade armas, juzgado, sentenciado y fusilado, se le considera heroe nacional. La poblacion deSan Miguel el Grande fue elevada al grado de ciudad el 8 de marzo de 1826, cambiando elnombre de la misma por “San Miguel de Allende” en honor al General Allende.

Para la decada de 1900, San Miguel de Allende estuvo a punto de convertirse en unpueblo fantasma. En 1926 fue declarado monumento historico por el Gobierno mexicano,por lo que desde entonces el desarrollo en el distrito historico esta restringido para conservarel caracter colonial del pueblo.

La fiesta de San Miguel Arcangel (29 de septiembre) es la celebracion mas importantepara la Ciudad de San Miguel de Allende, ya que se celebra al Santo Patrono de la ciu-dad: San Miguel Arcangel. Se lleva a cabo durante el ultimo fin de semana de septiembre.Comienza con la tradicional alborada y termina con una procesion con la imagen de SanMiguel Arcangel por las calles del centro de la ciudad y visitando los templos de las monjas,el oratorio y San Francisco para regresar a su parroquia. Tambien se llevan a cabo eventossociales, artısticos, deportivos, culturales ademas de sus famosas corridas de toros.

24 Olimpiada Iberoamericana de Matematicas, Mexico.

Page 24: 24 Olimpiada Iberoamericana de Matemáticas

Sitios de visita oficial

Guanajuato

La ciudad de Guanajuato esta ubicada en el centro de Mexico, es la capital del estado deGuanajuato, aproximadamente a 2,050 m sobre el nivel del mar. La ciudad se encuentradentro de un barranco rodeado por montanas, entre las cuales se encuentran los cerros deSan Miguel del Cuarto. Su nombre original, en lengua tarasca, significa lugar montuoso

de ranas. Cuenta con numerosas minas de oro y plata que han sido explotadas por lospobladores locales desde hace mas de 500 anos.

Posee importantes edificios coloniales, entre los que destacan el templo de la Companıa, elhospital de Marfil, la iglesia de San Francisco, la Alhondiga de Granaditas, construida comogranero publico en 1798, y la Casa de Diego Rivera. Guanajuato se fundo en 1557 bajo laautoridad del superintendente de minas Peratan de Rivera, que llamo al lugar Santa Fe yReal de Minas de Guanajuato. Adquirio la condicion de ciudad en 1741. Fue uno de los cen-tros del movimiento revolucionario para obtener la independencia de Espana, que comenzoen 1810. Fue sitiada por las fuerzas independentistas de Miguel Hidalgo y Costilla y, poste-riormente, se acuno en esta ciudad la primera moneda insurgente, en 1810, al crearse la Casade Moneda. En 1858, se establecio como capital provisional de la republica bajo el gobiernode Benito Juarez durante la guerra de los Tres Anos. Actualmente, es una de las ciudadesde arquitectura colonial mas importante del paıs y es sede, desde 1972, del Festival Interna-cional Cervantino, uno de los acontecimientos culturales de mayor relevancia en el ambitoiberoamericano. El centro historico de la ciudad y las minas adyacentes fueron declarados,en 1988, Patrimonio Cultural de la Humanidad. Poblacion (2000), 141,196 habitantes.

CIMAT

El Centro de Investigacion en Matematicas, A.C. (CIMAT) fue fundado en la ciudad deGuanajuato en 1980 y forma parte del Sistema de Centros Publicos CONACyT. El CIMATse orienta hacia la investigacion cientıfica bajo esquemas de excelencia, que lo han llevado ha-cia la integracion de una masa crıtica con grupos de alto rendimiento cientıfico, volviendoseun polo de desarrollo en creciente consolidacion. Asimismo esta dedicado a la generacion,transmision y aplicacion de conocimientos en sectores especializados, ası como a la for-macion de recursos humanos de alto nivel de excelencia en las areas de Matematicas Basicas,

202020

Page 25: 24 Olimpiada Iberoamericana de Matemáticas

Bernal 21

Probabilidad y Estadıstica y Ciencias de la Computacion. A 24 anos de su creacion, elCIMAT constituye uno de los centros de investigacion mas importantes del paıs. Su cons-tante busqueda por alcanzar el equilibrio entre las matematicas basicas y las aplicadas -aunada a las labores de vinculacion y transferencia tecnologica con los sectores productivoy social - otorgan al CIMAT un caracter muy singular. El CIMAT cuenta con personalacademico de alto nivel (mas del 80% de los investigadores del CIMAT pertenece al SistemaNacional de Investigadores y mas del 93% tiene doctorado) dedicado a las actividades deinvestigacion de frontera, de formacion de recursos humanos en los niveles de licenciatura yposgrado, y de vinculacion con los sectores social y productivo. Academicamente el Centroesta organizado en tres areas: Matematicas Basicas, Probabilidad y Estadıstica y Cienciasde la Computacion.

Bernal

Villa de San Sebastian Bernal mejor conocido como Bernal, es un bello pueblo colonialfundado en el siglo XVII, en el ano 1647, situado a 40 minutos de Queretaro y dos horas ymedia de la ciudad de Mexico.

Es conocido por su enorme monolito de roca maciza, (Pena de Bernal); la tercera masalta en el planeta despues del penon de Gibraltar en el mediterraneo y el Pan de Azucaren Rıo de Janeiro. La Villa de Bernal se ubica dentro del Municipio de Ezequiel Montes apocos minutos del Municipio de Colon y Cadereyta los cuales ofrecen al visitante atractivosque vale la pena descubrir.

Aparte del enorme atractivo de la pena, es un agasajo para aquellos que gustan de unverdadero lugar de descanso y disfrutar de la magia que se contagia y se respira en los rinconesde este bello lugar. Es famoso tambien porque miles de personas acuden al equinoccio deprimavera a cargarse de energıa.

Recientemente la Villa de Bernal adquirio el tıtulo de Pueblo Magico. Los PueblosMagicos son reconocidos de esta forma por ser localidades que tiene atributos simbolicos,leyendas, historia, hechos trascendentes, cotidianidad, en fin, magia que emana en cada unade sus manifestaciones socio culturales.

Tequisquiapan

Pasar un buen fin de semana en Tequisquiapan definitivamente se antoja. Recorrer suscallejones, disfrutar de su clima con un buen chapuzon y pasear por sus tiendas de artesanıas,es algo que no podemos dejar de hacer. Las propiedades del agua de este lugar han alcanzadola fama y su agradable clima permite disfrutar de las albercas del lugar. Por la tarde, se puedesalir a comer en uno de los riquısimos restaurantes de Tequisquiapan. Despues de la comida,no hay como caminar por las calles y calejones de este tıpico y florido lugar. En el recorrido seencuentran muchas tiendas de artesanıa. Uno puede encontrar desde ropa de piel, zapatos yartesanıa fina, hasta dulces tıpicos de la region y canastas en todo tipo de tejidos, tamanos y

24 Olimpiada Iberoamericana de Matematicas, Mexico.

Page 26: 24 Olimpiada Iberoamericana de Matemáticas

22 Sitios de visita oficial

formas. Entre otra de las actividades que puedes encontrar en Tequisquiapan esta el tranvıaturıstico para conocer un poco mas de este bello lugar. Bajando del tranvıa se puede visitarla casa de cultura y ver una exposicion o seguir el recorrido a pie. Tequisquiapan se encuentraa aproximadamente 40 minutos de la Ciudad de Queretaro, a 15 minutos de San Juan delRıo y a una hora y media de la Ciudad de Mexico.

24 Olimpiada Iberoamericana de Matematicas, Mexico.

Page 27: 24 Olimpiada Iberoamericana de Matemáticas

232323

Page 28: 24 Olimpiada Iberoamericana de Matemáticas

24 Programa 24 OIM

Programa 24 OIM

Lıderes —

Observadores A

Alumnos Tutores —

Observadores B

Miercoles 16 Llegada Llegada Llegada

Jueves 17

9:00 - 13:30 Simposio Simposio Simposio

16:00 - 19:30 Simposio Simposio Simposio

Viernes 18

9:00 - 13:30 Simposio Simposio Simposio

16:00 - 19:30 Simposio Simposio Simposio

21:30 Salida a San Miguel

de Allende

23:00 Entrega Lista corta

de problemas

Sabado 19

8:30 - 9:00 Lista corta de

problemas con

soluciones

9:00 - 14:00 Estudio de problemas Paseo por

Tequisquiapan

Paseo por

Tequisquiapan

16:00 - 20:00 Primera Reunion de

Jurado

Paseo por

Tequisquiapan

Paseo por

Tequisquiapan

Domingo 20

9:00 - 14:00 Segunda Reunion de

Jurado

11:00 - 15:00 Parque Bicentenario Parque Bicentenario

16:00 - 20:00 Tercera Reunion de

Jurado

Tiempo libre en Hotel Tiempo libre en Hotel

18:00 Registro a la OIM

Lunes 21

9:00 - 13:30 Cuarta Reunion de

Jurado

9:00 - 14:00 Paseo por Queretaro Paseo por Queretaro

17:00 - 19:00 Ceremonia de

Inauguracion

Ceremonia de

Inauguracion

Ceremonia de

Inauguracion

24 Olimpiada Iberoamericana de Matematicas, Mexico.

Page 29: 24 Olimpiada Iberoamericana de Matemáticas

Programa 24 OIM 25

Lıderes —

Observadores A

Alumnos Tutores —

Observadores B

Martes 22

7:00 Traslado a Queretaro

8:00 Traslado al Examen

9:00 - 11:00 Contestar preguntas

del examen

9:00 - 13:00 Examen Resolucion de

examen en Hotel

11:00 - 14:00 Paseo por Queretaro

13:00 Traslado al examen

16:00 Traslado a San

Miguel de Allende

Tiempo libre en Hotel Tiempo libre en Hotel

18:00 - 21:00 Reunion de Jurado

con Coordinadores

22:00 Entrega del 1er

examen

Miercoles 23

7:00 Traslado definitivo a

Queretaro

8:00 Traslado a examen

9:00 - 11:00 Contestar preguntas

del examen

9:00 - 13:00 Examen Resolucion del

examen en Hotel

11:00 - 11:30 Registro en el Hotel

13:00 Traslado al examen

11:30 - 18:00 Correcion de Examen

16:00 - 20:00 Paseo a Bernal Correccion de examen

18:00 Entrega de 2do

examen

Entrega de 2do

examen

18:00 - 20:00 Correccion de examen Correcion de examen

Jueves 24

8:00 - 12:00 Correccion de examen Correccion de examen

8:00 Salida a Rancho

Ahuizote

10:00 - 19:00 Rally

12:00 - 14:00 Coordinaciones Coordinaciones

16:00 - 20:00 Coordinaciones Coordinaciones

21:00 - 22:00 Reunion de Cortes Reunion de Cortes

24 Olimpiada Iberoamericana de Matematicas, Mexico.

Page 30: 24 Olimpiada Iberoamericana de Matemáticas

26 Programa 24 OIM

Viernes 25 Lıderes —

Observadores A

Alumnos Tutores —

Observadores B

8:00 Salida a Guanajuato Salida a Guanajuato Salida a Guanajuato

10:00 Llegada a la Mina Llegada a la Mina Llegada a la Mina

14:00 - 16:00 Comida en el CIMAT Comida en el CIMAT Comida en el CIMAT

16:00 - 19:00 Paseo al Centro de

Guanajuato

Paseo al Centro de

Guanajuato

Paseo al Centro de

Guanajuato

19:00 - 21:00 Cena en Guanajuato Cena en Guanajuato Cena en Guanajuato

21:00 Salida a Queretaro Salida a Queretaro Salida a Queretaro

Sabado 26

8:00 - 14:00 Manana libre Manana libre Manana libre

14:00 - 16:00 Comida en el Hotel Comida en el Hotel Comida en el Hotel

17:00 - 20:00 Ceremonia de

Clausura

Ceremonia de

Clausura

Ceremonia de

Clausura

20:00 - 24:00 Cena de Despedida Cena de Despedida Cena de Despedida

Domingo 27

8:00 Desayuno Desayuno Desayuno

10:00 Despedida Despedida Despedida

24 Olimpiada Iberoamericana de Matematicas, Mexico.

Page 31: 24 Olimpiada Iberoamericana de Matemáticas

Ceremonia de inauguracion 24 OIM

CEREMONIA DE INAUGURACINLunes 21 de septiembre de 2009, 17:00 horas

Teatro de la Republica

Apertura de la ceremonia y presentacion de autoridades.

Palabras de bienvenida por parte de la Dra. Radmila Bulajich Manfrino, Presidenta delComite Organizador de la IMO.

Palabras del Dr. Fernando Brambila Paz, presidente de la Sociedad Matematica Mexicana.

Presentacion de las Delegaciones (21).

Palabras del Dr. Rodolfo Tuiran, Subsecretario de Educacion Superior, representante delC. Felipe Calderon Hinojosa, Presidente Constitucional de los Estados Unidos Mexicanos.Inauguracion Oficial de la 24 Olimpiada Iberoamericana de Matematicas.

Evento Musical: Presentacion del Conjunto Mexico Folklorico del S.N.T.E., bajo la direcciondel Prof. Roberto Angeles Arellano.

Despedida.

Maestro de Ceremonias: Dr. Ignacio Barradas

Discurso bienvenida: Dra. Radmila Bulajich Manfrino

Honorables miembros del Presidium, concursantes y colegas,

Para todos nosotros es un honor compartir con ustedes la vigesimo cuarta edicion de laOlimpiada Iberoamericana de Matematicas. Antes quiero darles la mas cordial bienvenidaa esta bella ciudad de Queretaro, en la cual estamos realizando la mayor parte de nuestrasactividades.

272727

Page 32: 24 Olimpiada Iberoamericana de Matemáticas

28 Ceremonia de inauguracion 24 OIM

El esfuerzo realizado por un pequeno grupo de paıses hace 24 anos se ve ahora plasmadoen el gran numero de paıses que participan actualmente y en la gran cantidad de jovenesque se han involucrado en esta organizacion.

Todos sabemos que el aprendizaje de las matematicas plantea uno de los mayores retospara el mundo civilizado, ası ha sido en el pasado y lo sigue siendo en el presente. Porello crear, disenar, promover y sostener espacios donde impulsar la creatividad latente ennuestros jovenes debe ser un objetivo fundamental. La olimpiada de matematicas es unode estos espacios. Miles de estudiantes participan cada ano y en cada paıs en alguna de lasetapas de las olimpiadas: concursos estatales, nacionales o internacionales. Posteriormente,regresan a su lugar de origen a recrear la experiencia vivida.

En Mexico, muchos de ellos se han integrado a la planta docente del paıs y han contribuidoa formar a muchas generaciones de estudiantes. Hoy algunos estan aquı presentes, comoparticipantes, miembros del jurado y organizadores.

Los problemas que enfrentan los concursantes en estas competencias reclaman conocimien-tos, razonamientos y estrategias adecuadas para encontrar su solucion. El pensamiento logicoque se pone en juego es la respuesta a los constantes desafıos intelectuales, y tan generoso es,que se convierte en una capacidad para enfrentar cualquier tipo de problemas, incluso los deorden practico. Hoy no se puede ser sociologo, musico o biologo sin saber utilizar, aunque seatangencialmente, las matematicas. Las matematicas estan presentes en todo el mundo quenos rodea, sin matematicas no existirıan todos los medios de comunicacion que actualmenteconocemos como las computadoras, el internet, la telefonıa etc. Los aviones no volarıan, losautomoviles no caminarıan. La pintura y la escultura se verıan afectadas sin el uso de lageometrıa, donde la belleza de las proporciones tiene raıces puramente matematicas. Estantambien presentes en la musica. Termino aquı la lista no porque no las encontremos en otrasdisciplinas simplemente porque la lista es demasiado larga. En fin? dejo de lado nuestrainclinacion a cantar loas a las matematicas para agradecer a todas las instituciones que hacenposible que eventos de este tipo se puedan llevar a cabo y hago un llamado a realizar uncambio de fondo en las polıticas educativas, de esto depende nuestro futuro.

Agradezco de manera muy especial a Carmen Sosa y su equipo, que trabajaron durantetodo el ano y a todo el comite organizador. A la Universidad Autonoma de Queretaro, a laSecretarıa de Educacion Publica del Estado y a la federal, al Consejo de Ciencia y Tecnologıade Queretaro y al Nacional, a la Universidad de Guanajuato, al Centro de Investigacion enMatematicas y a todas las instituciones que nos apoyaron.

Para terminar, les deseo mucha suerte a los participantes y los invito a trabajar, en elfuturo, para mantener viva a la Olimpiada Iberoamericana de Matematicas.

24 Olimpiada Iberoamericana de Matematicas, Mexico.

Page 33: 24 Olimpiada Iberoamericana de Matemáticas

Carta de la reunion del CEA

CONSTANCIA

El Jurado Internacional de la 24 Olimpiada Iberoamericana de Matematica, conformadopor los lıderes de los siguientes paıses: Argentina, Bolivia, Brasil, Chile, Colombia, CostaRica, Cuba, Ecuador, El Salvador, Espana, Guatemala, Honduras, Mexico, Nicaragua,Panama, Paraguay, Peru, Portugal, Puerto Rico, Uruguay y Venezuela, reunido en Santiagode Queretaro, Mexico, a los 21 dıas del mes de septiembre de 2009, designa como miembrosdel Comite Ejecutivo Asesor de la OIM (CEAOIM), a los siguientes representantes:

Presidente Gabriela Gomez Pasquali (Representante de Paraguay)Secretario Radmila Bulajich Manfrino (Representante de Mexico)Vocal Carlos Yuzo Shine (Representante de Brasil)Vocal Mario Marın Sanchez (Representante de Costa Rica)Vocal Mayra Castillo (Representante del Jurado por Guatemala)

Este comite tendra vigencia desde el 26 de septiembre de 2009, hasta la primera reuniondel Jurado de la 25 Olimpiada Iberoamericana de Matematica. Se confirma ademas, lossiguientes paıses para realizar las proximas ediciones de la Olimpiada Iberoamericana deMatematica:

Ano 2010 - ParaguayAno 2011 - Costa Rica

Radmila Bulajich ManfrinoVicepresidente del Jurado

Jose Antonio Gomez OrtegaPresidente del Jurado

292929

Page 34: 24 Olimpiada Iberoamericana de Matemáticas

Participantes

ArgentinaLıder Patricia FauringTutor Flora GutierrezGuıa Gustavo Adolfo Ramos PalaciosARG1 German Ezequiel StefanichARG2 Ivan Sadofschi CostaARG4 Juan Andres DodykARG3 Miguel Sebastian Maurizio

BoliviaLıder Gustavo Michel GarciaTutor Sonia Ximena Nivia Cordero CardenasGuıa Emilio Angulo PerkinsBOL2 Alvaro Ruben Hurtado MaldonadoBOL4 Antonio Santiago Penaranda HuancaBOL3 Daniela Alejandra Lazcano Gonzales

BrasilLıder Onofre Campos da Silva FariasTutor Luzinalva Miranda de AmorimGuıa Alejandro Luis Gonzalez TokmanBRA1 Marcelo Tadeu de Sa Oliveira SalesBRA2 Marco Antonio Lopes PedrosoBRA3 Matheus Secco Torres da SilvaBRA4 Renan Henrique Finder

303030

Page 35: 24 Olimpiada Iberoamericana de Matemáticas

Participantes 31

ChileLıder Hernan Burgos VegaTutor Hugo CaerolsGuıa Mariano Raul Gonzalez TokmanCHL4 Andrea ChaniqueCHL1 Anibal VelozoCHL3 Felipe GarciaCHL2 Valentina Toro

ColombiaLıder Juan Ignacio RestrepoTutor Pedro Luis BarriosGuıa Orlando Ochoa CastilloCOL4 Carlos Andres Dıas CarrilloCOL2 David Mauricio Arcila PardoCOL1 Jorge Alberto Olarte ParraCOL3 Nicolas Eduardo Del Castillo Munoz

Costa RicaLıder Jeremıas Ramırez JimenezTutor Andres Alonso Segura MedinaGuıa Edgar Gonzalez ArreolaCRI4 German Jose Mora SaenzCRI1 Jonathan Valverde LizanoCRI3 Julio Andres Vargas RamırezCRI2 Ruben Rodrıguez Roman

CubaLıder Mario Dıaz GonzalezGuıa Juan Antonio Villeda ResendizCUB4 Daniel Otero BaguerCUB2 Jorge Estrada HernandezCUB3 Jose Moraguez PinolCUB1 Reynaldo Gil Pons

24 Olimpiada Iberoamericana de Matematicas, Mexico.

Page 36: 24 Olimpiada Iberoamericana de Matemáticas

32 Participantes

EcuadorLıder Luis Rodrıguez OjedaTutor Janet Patricia ValdiviezoGuıa Jose Trinidad Barajas VegaECU1 Christian Fernando Abad CoronelECU3 Gabriel Isaac Bravo BarahonaECU2 Jennifer Stephania Figueroa Mendoza

ECU4 Miguel Angel Ordonez Mera

El SalvadorLıder Eder Alexander Jacobo ArevaloTutor Eduardo Arnoldo Aguilar CanasGuıa Antonio de Jesus Torres HernandezSLV1 Hector Enmanuel Alberti ArroyoSLV3 Jaime Antonio Bermudez HuezoSLV2 Julio Cesar Ayala MenjivarSLV4 Nahomy Jhopselyn Hernandez Cruz

EspanaLıder Salvador Villegas BarrancoTutor Daniel Lasaosa MedardeGuıa Ivonne del Carmen Colorado ChairezESP4 Ander Lamaison VidarteESP3 Glenier Lazaro Bello BurguetESP2 Ivan Geffner FuenmayorESP1 Moises Herradon Cueto

GuatemalaLıder William Roberto Gutierrez HerreraTutor JoseCarlos Alberto Bonilla AldanaGuıa Josue Hernandez CarlosGTM1 Alejandro Jose Vargas De LeonGTM2 Fernando Jose Mazariegos CamasGTM4 Jose Alberto Ligorrıa TaracenaGTM3 Rafael Alejandro Martınez Marquez

24 Olimpiada Iberoamericana de Matematicas, Mexico.

Page 37: 24 Olimpiada Iberoamericana de Matemáticas

Participantes 33

HondurasLıder Luis Armando Ramos PalaciosTutor Angel Roberto Rivera MunozGuıa Pablo Luis Pena DıazHND1 Jose Ramon MadridHND2 Nestor Alejandro BermudezHND4 Oscar Hernan Madrid PadillaHND3 Sergio David Manzanarez

MexicoLıder Alejandro Bravo MojicaTutor Fernando Campos GarcıaObservador A Denisse Segovia Rongel

Guıa Julio Cesar Iniguez AlonsoMEX4 Cesar Daniel Bibiano VelascoMEX2 Daniel Perales AnayaMEX1 Erik Alejandro Gallegos BanosMEX3 Manuel Guillermo Lopez Buenfil

NicaraguaLıder Carlos Jose Walsh MendozaTutor Hank De Jesus Espinoza Serrano

Guıa Teresa de Jesus Iniguez AlonsoNIC4 Hans Mijail Castro SozaNIC3 Melvin Enoc Chavarrıa ZelayaNIC1 Rufo Holvin Casco ArevaloNIC2 Silvio Francisco Duarte Vallejos

PanamaLıder Pedro A. Marrone GaudianoTutor Alexander Neblett AlvaradoGuıa Hector Daniel Banos CervantesPAN1 Antonio Fan HuangPAN2 Edwin Hernandez AnguizolaPAN3 Ricardo Wong Yau

24 Olimpiada Iberoamericana de Matematicas, Mexico.

Page 38: 24 Olimpiada Iberoamericana de Matemáticas

34 Participantes

ParaguayLıder Gabriela Maria Ratti BittingerTutor Paola Marıa Camila Villalba FioreObservador A Cesar Talavera G.Observador A Gabriela Gomez PasqualiGuıa Abraham Solis Garcıa InaPRY2 Edgar Federico Elizeche ArmoaPRY3 Nair Isabel Aguilera MartınezPRY4 Osmar Mateo Quinonez VazquezPRY1 Rodrigo Daniel Benıtez Maidana

PeruLıder Jorge Joel Tipe VillanuevaTutor Claudio Vicente Espinoza ChoqquepuraGuıa Luis Alberto Barradas ChaconPER2 Amilcar Enrique Velez SalamancaPER1 Julian Alonso Mejıa CorderoPER4 Percy Guerra RiosPER3 Ricardo Jesus Ramos Castillo

PortugalLıder Joel Pedro de Oliveira MoreiraTutor Joao Nuno Mestre Fernandes Silva

Guıa Laura Cecilia Avila JaureguiPRT1 Frederico Oliveira ToulsonPRT2 Joao Miguel Magalhaes dos SantosPRT3 Jorge Ricardo Landeira da Silva MirandaPRT4 Pedro Manuel Passos de Sousa Vieira

Puerto RicoLıder Luis CaceresTutor Yuri RojasObservador A Juan RomeroObservador B Arturo PortnoyGuıa Paulina Tello MotaPRI4 Alan Wagner RodrıguezPRI1 Aravind ArunPRI3 Eric Crespo LezamaPRI2 George Arzeno Soltero

24 Olimpiada Iberoamericana de Matematicas, Mexico.

Page 39: 24 Olimpiada Iberoamericana de Matemáticas

Participantes 35

UruguayLıder Leonardo Lois Da SilvaTutor Fabian KozynskiGuıa Gilberto Sanchez de la IslaURY3 Ismael Valentın Rodrıguez BrenaURY2 Juan Pablo GarciaURY1 Martin CottoURY4 Matıas Escuder Rebori

VenezuelaLıder Henry Martınez LeonTutor Eduardo SarabiaGuıa Edwin Alain Lopez RomoVEN1 Carmela Acevedo CastilloVEN2 Mauricio Marcano SandovalVEN3 Tomas Rodrıguez Oramas

24 Olimpiada Iberoamericana de Matematicas, Mexico.

Page 40: 24 Olimpiada Iberoamericana de Matemáticas

Problemas y Soluciones de la 24 OIM

Paıses que mandaron problemas

Problema 1 - Mexico - Efren Perez TerrazasProblema 2 - Mexico - Juan Jose Alva GonzalezProblema 3 - El Salvador - Eder Alexander Jacobo ArevaloProblema 4 - Mexico - Eduardo Velasco BarrerasProblema 5 - Argentina - Equipo ArgentinoProblema 6 - Argentina - Equipo Argentino

Problema 1 Mexico

Sea n un natural mayor que 2. Supongamos que n islas estan ubicadas en un cırculo y queentre cada dos islas vecinas hay dos puentes como en la figura:

x3xn

x2x1

xn−1 xj

Comenzando en la isla x1, ¿de cuantas maneras se pueden recorrer los 2n puentes pasandopor cada puente exactamente una vez?

Solucion. Es claro que existe simetrıa entre comenzar el recorrido “en el sentido opuestoal de las manecillas del reloj” (sentido anti-horario o levogiro), o bien comenzarlo “en elsentido de las manecillas del reloj”(sentido horario o dextrogiro). Por consiguiente, bastara

363636

Page 41: 24 Olimpiada Iberoamericana de Matemáticas

Problemas y Soluciones de la 24 OIM 37

con conocer cuantos caminos hay en la grafica que recorren todas las aristas y cada una deellas una sola vez, que empiezan en x1 y que recorren su primer arista en el sentido horarioy al resultado multiplicarlo por 2Se denotara por x2 al vertice que se encuentra al final de la primera arista y por x3, . . . , xn alos demas vertices que se van encontrando al recorrer la circunferencia en el sentido horario.

x3xn

x2x1

xn−1 xj

Se dice que xj es un vertice de rebote (o bien, que hay un rebote en xj) si desde x1 se avanzahasta xj y luego se regresa a x1 por la direccion opuesta, es decir que para 1 < j ≤ nhay un rebote en xj si el camino da origen a la sucesion de vertices x1 − x2 − · · · − xj−1 −xj − xj−1 − · · · − x2 − x1 y si el rebote es en x1 entonces se tiene una sucesion de verticesx1 − x2 − · · · − xn − x1 − xn − · · · − x2 − x1.Es inmediato que hay

2 × 2 × 2 × · · · × 2︸ ︷︷ ︸j−1

× 1 × 1 × · · · × 1︸ ︷︷ ︸j−1

= 2j−1

caminos desde x1 hasta x1 determinados por el vertice de rebote xj , pues hay dos aristaspor elegir en cada vertice xi con i ∈ {1, . . . , j − 1} al desplazarse hasta xj , mientras que enel desplazamiento de regreso solamente queda una opcion en cada vertice.Si xj es un vertice de rebote entonces, para poder recorrer todo la grafica pasando una solavez por cada arista, es necesario que ahora se realice un camino hasta xj en el sentido anti-horario y luego un camino de regreso hasta x1 en el sentido horario, es decir que ahora hayuna sucesion de vertices x1 − xn − xn−1 − · · ·− xj+1 − xj − xj+1 − · · ·− xn−1 − xn − x1, parala cual hay

2 × 2 × 2 × · · · × 2︸ ︷︷ ︸n−j+1

× 1 × 1 × · · · × 1︸ ︷︷ ︸n−j+1

= 2n−j+1

caminos.Por consiguiente hay n (2n) caminos con rebote que comienzan en x1 y que su primer aristase recorre en el sentido horario.

24 Olimpiada Iberoamericana de Matematicas, Mexico.

Page 42: 24 Olimpiada Iberoamericana de Matemáticas

38 Problemas y Soluciones de la 24 OIM

Solamente queda considerar el caso en que no hay rebotes, es decir que hay una sucesion devertices x1 − x2 − · · · − xn − x1 − x2 − · · · − xn − x1, para la cual tambien hay 2n caminosposibles.Se concluye que existen 2n(n+1) caminos que empiezan en x1 y cuya primer arista se recorreen el sentido horario, por lo que la respuesta es

2 · 2n (n + 1) = 2n+1 (n + 1) .

Problema 2 MexicoPara cada entero positivo n se define an = n + m donde m es el mayor entero tal que

22m ≤ n2n. Determinar que enteros positivos no aparecen en la sucesion an.

Solucion. Realizando las cuentas, se ve que los primeros terminos de la sucesion son 1,3,5, 6, 7, 9, 10, 11, 12, 13, 14, 15, 17, . . ., por lo que se demostrara que la respuesta son losnaturales que no son potencias de 2.Depende del primer momento en que m cambia el que un numero aparezca o no en lasucesion. Ahora bien, en la lista mostrada, se ve que a2−1, a22−2 y a23−3 son los elementosde la sucesion previos a un “salto”.Si n = 2k − k, entonces n2n ≤ 22k

y log2 (n) < k. Luego

log2 (n) + n < k + 2k − k = 2k,

y n2n < 22k

, por lo quea2k−k < 2k. (1)

Si n = 2k−k+1, entonces 22k

≤ n2n. Como 2k−1 > k−1 se tiene que 2k−k+1 > 2k−1, y asılog2 (n) > k−1. Se sigue de las identidades previas que n+log2 (n) > k−1+2k−k+1 = 2k,luego n2n > 22k

, por lo quea2k−k+1 ≥ 2k + 1. (2)

Es facil ver que si 22k

> n2n, entonces 22k+1

> (n + 1) 2n+1, ası que an+1 − an ≤ 2. De esteargumento y de las identidades (1) y (2) se obtiene que

a2k−k = 2k − 1;

a2k−k+1 = 2k + 1. (3)

Mas aun, como an+1 − an ≥ 1, las identidades (3) muestran que a2k−k+t = 2k + t parat ∈

{1, . . . , 2k − 1

}. Si no, de otra manera se tendrıa que a2k+1−(k+1) = a2k−k+2k−1 > 2k+1−1.

Se concluye que en la sucesion aparecen todos los enteros positivos excepto los que sonpotencias de 2.

Segunda Solucion Los primeros terminos de la sucesion son 1, 3, 5, 6, 7, 9, 10, 11, 12, 13,14, 15, 17, . . . , esto sugiere que la sucesion omite las potencias de 2 (mayores que 1).

24 Olimpiada Iberoamericana de Matematicas, Mexico.

Page 43: 24 Olimpiada Iberoamericana de Matemáticas

Problemas y Soluciones de la 24 OIM 39

Sea bn la sucesion formada por los enteros positivos que no son de la forma 2k con k ≥ 1ordenados de manera creciente. Se mostrara que bn = an para todo n ≥ 1.Para n = 1 tenemos que b1 = 1 = a1.Ahora se considera n ≥ 2. Sea k el entero positivo tal que 2k ≤ n < 2k+1. Como n ≥ 2 yk es positivo, 2k < n + k < n + k + 1 < 2k+1 + k + 1 < 2k+2. Esto muestra que n + k estaentre las potencias 2k y 2k+2. Hay dos casos:

1. 2k < n + k < 2k+1

Esto implica que n + k + 1 ≤ 2k+1. De los n + k enteros del conjunto {1, 2, . . . , n + k}exactamente k de ellos son potencias de 2: 21, 22, . . . , 2k (pues 2k+1 > n + k), por loque los restantes n enteros son los primeros n terminos de la sucesion bj . Esto dice quebn es el ultimo entero del conjunto {1, 2, . . . , n + k} que no es potencia de 2. Pero eneste caso que se analiza, n + k no es potencia de 2, por lo que bn = n + k.

Por otra parte, de las desigualdades que satisfacen n y k, tenemos que 22k

< 2n+k =2k2n ≤ n2n < 2k+12n = 2n+k+1 ≤ 22k+1

. Entonces k es el mayor entero tal que22k

≤ n2n, por lo que an = n + k y entonces an = bn.

2. 2k+1 ≤ n + k < 2k+2

En este caso sucede que 2k+1 ≤ n+k < n+k +1 < 2k+2. Aquı de los n+k +1 enterospositivos del conjunto {1, 2, . . . , n + k + 1} exactamente k + 1 de ellos son potenciasde 2, por lo que los restantes n son los primeros n terminos de la sucesion bj . Luegobn es el ultimo entero del conjunto {1, 2, . . . , n + k + 1} que no es potencia de 2. Peron + k + 1 no es potencia de 2, por lo que bn = n + k + 1.

Por otra parte, de las desigualdades que satisfacen n y k, tenemos que 22k+1 ≤ 2n+k =2k2n ≤ n2n < 2k+12n = 2n+k+1 < 22k+2

. Entonces k + 1 es el mayor entero tal que22k+1

≤ n2n, por lo que an = n + k + 1 = bn.

Problema 3 El SalvadorSean C1 y C2 dos circunferencias de centros O1 y O2 con el mismo radio, que se cortan en Ay en B. Sea P un punto sobre el arco AB de C2 que esta dentro de C1. La recta AP corta aC1 en C, la recta CB corta a C2 en D y la bisectriz de 6 CAD intersecta a C1 en E y a C2 enL. Sea F el punto simetrico a D con respecto al punto medio de PE. Demostrar que existeun punto X que satisface 6 XFL = 6 XDC = 30◦ y CX = O1O2.

Solucion. Observemos primero que, puesto que F es el simetrico de D con respecto al puntomedio de PE, se tiene que las diagonales del cuadrilatero FPDE se intersectan en su puntomedio. Por tanto, FPDE es un paralelogramo. Ası que FP = ED.Ahora bien, como las circunferencias son iguales y AB es la cuerda comun, AB abre elmismo arco en ambas circunferencias. De aquı que 6 ACB = 6 ADB. Entonces el trianguloACD es isosceles. Como AL es bisectriz del 6 CAD, se tiene que AL es la mediatriz de CD.

24 Olimpiada Iberoamericana de Matematicas, Mexico.

Page 44: 24 Olimpiada Iberoamericana de Matemáticas

40 Problemas y Soluciones de la 24 OIM

Ademas, puesto que 6 CAE = 6 LAD, las cuerdas CE y LD son iguales. Observemos que6 BCA = 6 BAE = 6 BDL por abrir arcos iguales. Concluimos que CE es igual y paralelaa DL. Ası que ECLD es un paralelogramo. De aquı que CL es igual y paralela a DE.Por tanto, CL es igual y paralela a FP . Hemos demostrado que FPCL tambien es unparalelogramo.Como CD es perpendicular a EL, EDCL es un rombo. Ası que ED = DL = LC = CE.Observemos que PL = LD pues 6 PAL = 6 LAD. Concluimos que FC = PL = LD =LC = FP . Puesto que LD = CE y las circunferencias son iguales, los triangulos O1CE yO2LD son congruentes. Como LD es paralela a CE concluimos que O1C es paralela O2L.Esto muestra que O1O2LC es un paralelogramo. De aquı que O1O2 = CL = LD = FC.Ahora bien, sea X un punto tal que el triangulo LCX es equilatero. Como CF = CL = CX,C es el circuncentro de FLX. De aquı que 6 LFX = 1

26 LCX = 1

260◦ = 30◦. Hemos mostrado

que X cumple lo que se pide.

Problema 4 MexicoSea ABC un triangulo con AB 6= AC. Sean I el incentro de ABC y P el otro punto deinterseccion de la bisectriz exterior del angulo A con el circuncırculo de ABC. La rectaPI intersecta por segunda vez al circuncırculo de ABC en el punto J . Demostrar que loscircuncırculos de los triangulos JIB y JIC son tangentes a IC y a IB, respectivamente.

Solucion. Sea M el punto de intesteccion de AI con el circuncırculo de ABC. Como AMy AP son la bisectriz interior y exterior del angulo A, entonces forman un angulo recto enA. Por tanto, 6 PMA = 90◦ − 6 APM . Pero 6 APM = 6 ACM = 6 ACB + 6 BCM =6 C + 6 BAM = 6 C + 1

26 A. Concluimos que 6 PMA = 90◦ − 6 C − 1

26 A = 1

2( 6 B − 6 C).

Por otro lado, 6 BJP = 6 BCP = 6 BCA+ 6 ACP = 6 BCA+ 6 AMP = 6 C + 12( 6 B− 6 C) =

12( 6 B + 6 C) = 6 IBC + 6 ICB. Es decir, 6 BJP es igual al angulo externo en I del triangulo

IBC. Esto muestra que CI es tangente al circuncırculo del JIB. Analogamente, se tieneque BI es tangente al circuncırculo de JIC.

Segunda Solucion Como AP es bisectriz exterior del angulo en A, se tiene que P es elpunto medio del arco BC que contiene a A. Sean C1 la circunferencia tangente a CI enI que pasa por B y C2 la circunferencia tangente a BI en I que pasa por C. Sea K elsegundo punto de interseccion, distinto de I, de C1 con C2. Veamos que K = J . ComoC1 es tangente a CI, 6 BKI es igual al angulo exterior en I del triangulo BIC. Es decir,6 BKI = 6 IBC + 6 ICB = 1

2( 6 B + 6 C). Analogamente, 6 CKI = 1

2( 6 B + 6 C). Por tanto,

6 BKC = 6 BKI + 6 CKI = 2(12( 6 B + 6 C)) = 6 B + 6 C = 180◦ − 6 A. De aquı que el

cuadrilatero ABKC es cıclico. Entonces K pertenece al arco BC, del circuncırculo deltriangulo ABC, que no contiene a A. Ademas, puesto que 6 BKI = 1

2( 6 B + 6 C) = 6 CKI,

KI es bisectriz del angulo 6 BKC. Por lo tanto, KI pasa por el punto medio del arco BCopuesto a K. Es decir, K, I y P son colineales. Ası que K = J .

Tercera Solucion Observemos que, como AP es bisectriz exterior del angulo en A, entoncesP es el punto medio del arco BC que contiene a A. De aquı que 6 BJP = 6 PJC. Pero

24 Olimpiada Iberoamericana de Matematicas, Mexico.

Page 45: 24 Olimpiada Iberoamericana de Matemáticas

Problemas y Soluciones de la 24 OIM 41

6 BJP + 6 PJC = 6 BJC = 180◦− 6 A = 6 B+ 6 C. Por lo que 6 BJP = 6 PJC = 12( 6 B+ 6 C).

Ademas, el angulo exterior del 6 BIC mide 6 IBC + 6 ICB = 12( 6 B+ 6 C) = 6 BJP = 6 PJC.

El resultado se sigue por el teorema del angulo semi-inscrito.

Problema 5 ArgentinaLa sucesion an esta definida por

a1 = 1, a2k = 1 + ak y a2k+1 =1

a2k

, para todo entero k ≥ 1.

Demostrar que todo numero racional positivo aparece exactamente una vez en esta sucesion.

Solucion. Es claro que a2k > 1 y a2k+1 ∈ (0, 1) si k > 0. Ahora a1 = 1, a2 = 2, a3 = 1/2.Sea u/v un numero racional positivo con u y v coprimos. Se probara por induccion sobres = u + v que ak = u/v para algun k. Como se ha visto, esto se cumple para s = 2 ys = 3. Si u/v > 1 se considera al numero u/v − 1 = (u − v)/v. Observe que u − v y v soncoprimos y que la suma de ellos es (u − v) + v = u < s, por hipotesis de induccion se tieneque ak = (u − v)/v para algun k. Por lo tanto, a2k = 1 + ak = u/v. Si u/v < 1, entoncesv/u > 1. Por un argumento como el anterior, existe k tal que ak = v/u−1 = (v−u)/u. Porlo tanto, a2k = 1 + ak = v/u, y entonces a2k+1 = 1/a2k = u/v.Para ver que dado un racional positivo u/v hay solo un k tal que ak = u/v, supongase que noes ası y sea u/v un racional tal que ai = aj = u/v, i < j, con i mınimo con esta propiedad.Entonces, los dos indices i y j tienen la misma paridad (y los dos son pares si u/v > 1, ylos dos son impares si u/v < 1). Si son los dos pares, con i = 2i0, j = 2j0, tenemos queai = a2i0 = 1 + ai0 y aj = a2j0 = 1 + aj0 , por lo tanto ai0 = aj0, contradiciendo la manera deescoger i. Si son los dos impares con i = 1, entonces aj = 1, y esto puede pasar si y solo sij = 1. Si i > 1, entonces i = 2i0 + 1, j = 2j0 + 1, ai = 1/a2i0 , aj = 1/a2j0, y se obtiene quea2i0 = a2j0 , una vez mas se contradice la minimalidad de i.

Segunda Solucion Si se escribe la representacion binaria de n como

n = 2α0 + 2α0+α1 + · · ·+ 2α0+···+αs

donde s ≥ 0, α0 ≥ 0 y α1, . . . , αs positivos, se puede mostrar facilmente por induccion que

an = [α0, α1, . . . , αs−1, αs + 1], (4)

donde se utiliza la notacion [b0, b1, b2 . . . , bt] para la fraccion continua

b0 +1

b1 +1

b2 + · · ·+1

bt−1 +1

bt

. (5)

24 Olimpiada Iberoamericana de Matematicas, Mexico.

Page 46: 24 Olimpiada Iberoamericana de Matemáticas

42 Problemas y Soluciones de la 24 OIM

En efecto, si s = 0 se tiene

a2α0 = 1 + a2α0−1 = 2 + a2α0−2 = · · · = α0 + a1 = α0 + 1 = [α0 + 1],

lo que prueba la formula (4) para s = 0. Suponiendo que s > 0 y escribiendo m = 2α1 +2α1+α2 + · · ·+ 2α1+···+αs, se tiene que n = 2α0(1 + m). Como m es par, obtenemos

an = 1 + a2α0−1(1+m) = · · · = α0 + a1+m = α0 +1

am

.

Ahora la formula (4) se sigue facilmente por induccion sobre m, ya que el numero de suman-dos binarios de m es < s. Ahora la conclusion del problema se sigue del hecho conocidoque cada numero racional admite una representacion en fraccion continua como aquella queaparece en el lado derecho de (4) que ademas es unica ya que para s ≥ 0 se tiene que αs+1 > 1(sin esta condicion, cualquier numero racional admite de hecho dos representaciones comouna fraccion continua que son

[b0, . . . , bs−1, bs] = [b0, . . . , bs−1, bs − 1, 1]

para bs > 1).

Problema 6 ArgentinaAlrededor de una circunferencia se marcan 6000 puntos y cada uno se colorea con uno de 10colores dados, de manera tal que entre cualesquiera 100 puntos consecutivos siempre figuranlos 10 colores. Hallar el menor valor k con la siguiente propiedad: Para toda coloracion deeste tipo existen k puntos consecutivos entre los cuales figuran los 10 colores.Solucion. El menor valor es k = 89. Veamos primero que, en toda coloracion con lascaracteristicas del enunciado, los 10 colores estan presentes en algun arco de longitud 89.Aquı, y en lo que sigue, un arco de longitud ℓ significa ℓ puntos consecutivos alrededor dela circunferencia. Siempre supondremos que la direccion del movimiento alrededor de lacircunferencia es en el sentido del reloj.Supongamos por el contrario que para alguna coloracion, ningun arco de longitud 89 contienetodos los 10 colores. Entonces sucedera lo siguiente:

(i) en cada arco de longitud 11 hay un color que no figura entre los 89 puntos subsiguientes;

(ii) cada arco de longitud 12 contiene precisamente 2 colores.

Veamos porque. Si (i) es falso para algun arco de longitud 11, en los 89 puntos subsiguientestienen que estar presentes los 10 colores (pues en cada arco de longitud 100 figuran todoslos colores), lo que es contrario a lo supuesto. Para ver (ii), dividimos cualquier arco delongitud 100 en 9 arcos A1, . . . , A9, los primeros 8 de longitud 11 cada uno y A9 de longitud12. Por (i), para cada i = 1, . . . , 8 existe un color i en Ai que no figura en Ai+1, . . . , A9.

24 Olimpiada Iberoamericana de Matematicas, Mexico.

Page 47: 24 Olimpiada Iberoamericana de Matemáticas

Problemas y Soluciones de la 24 OIM 43

Claramente, los colores 1, . . . , 8 son distintos, luego hay a lo mas 10 − 8 = 2 colores en A9.En este argumento A9 puede ser cualquier arco de longitud 12 de la circunferencia.Entonces cada arco de longitud 12 contiene a lo mas 2 colores. En realidad contiene exacta-mente 2 colores. En efecto, si algun arco de longitud 12 tienen todos sus puntos del mismocolor, consideremos A junto con el arco B de longitud 88 que sigue a A. En el arco A ∪ Bfiguran los 10 colores de modo que B y el ultimo punto de A forman un arco de longitud89 en el que figuran los 10 colores, en contradiccion con nuestro supuesto. Entonces (ii) esverdadero.Ahora tomemos un punto p0 del color 0 que no figura en los siguientes 89 puntos. Tal puntoexiste, en virtud de (i). Los 11 puntos subsiguientes a p0 forman un arco A1. En el arco{p0} ∪ A1 de longitud 12, hay exactamente 2 colores, por (ii). Uno de ellos es el color 0;llamemos al otro color 1. Ademas, por la eleccion de p0, en A1 no figura el color 0. Luego todoel arco A1 es de color 1. Ahora, (i) garantiza que el color 1 no aparece en los subsiguientes89 puntos siguientes al ultimo punto p1 de A1.Exactamente el mismo argumento se aplica a p1 y al subsiguiente arco A2 de longitud 11.Entonces el arco A2 contiene un unico color, que ademas no figura entre los subsiguientes89 puntos. Aplicando repetidas veces el mismo razonamiento se ve que la circunferenciase divide en arcos de longitud 11 con todos los puntos del mismo color en cada arco. Sinembargo esto es imposible, pues el numero total de puntos, 6000, no es un multiplo de 11.Por lo tanto, hay un arco de longitud 89 en el que figuran los 10 colores.El siguiente ejemplo muestra que la longitud 88 no es suficiente para garantizar esto engeneral.Dados 10 colores denotados por 0, . . . , 9, tomamos los siguientes 10 arcos A0, . . . , A9, cadauno de longitud 12:

A0 = 000101101000, A1 = 111212212111, . . . , A8 = 888989989888,

A9 = 999090090999.

El arco Ai+1 se obtiene del arco Ai por desplazamiento cıclico i 7→ i + 1 de los colores.Repetimos el patron A0, . . . , A9, 50 veces para obtener una coloracion con 10 colores de los6000 puntos alrededor de la circunferencia. Esto es posible, pues 6000 = 50 · 120. Sea Acualquier arco de longitud 100 de esta coloracion. Por la periodicidad podemos suponer queA comienza con un punto de un arco A0, entonces contiene con certeza el color 0. LuegoA contiene los arcos completos A1, . . . , A7 que siguen a A0, y al menos cuatro puntos delarco A8. Como el cuarto punto de A8 tiene color 9, y como los colores 1, . . . , 8 figuran enA1, . . . , A7, concluimos que A contiene los 10 colores.Ahora tomemos cualquier arco B de longitud 88 y supongamos de nuevo que comienza enun punto de un arco A0. Consideremos los siguientes arcos A1, . . . , A8. El ultimo punto deB esta, o bien entre los 9 ultimos puntos de A7, o bien entre los primeros tres puntos deA8, que tienen color 8. Como A0, . . . , A7 no contienen el color 9, se infiere que tampoco locontiene el arco B. La demostracion esta completa.

24 Olimpiada Iberoamericana de Matematicas, Mexico.

Page 48: 24 Olimpiada Iberoamericana de Matemáticas

44 Problemas y Soluciones de la 24 OIM

Primeiro dia

Problema 1

Seja n um natural maior que 2. Suponhamos que n ilhas estejam localizadas ao redor de umcırculo e que entre cada duas ilhas vizinhas haja duas pontes, como na figura:

x3xn

x2x1

xn−1 xj

Partindo da ilha x1, de quantas maneiras se podem percorrer as 2n pontes passando porcada ponte exatamente uma vez?

Problema 2

Para cada inteiro positivo n definimos an = n + m, em que m e o maior inteiro tal que

22m ≤ n2n. Determinar quais inteiros positivos nao aparecem na sequencia an.

Problema 3

Sejam C1 e C2 duas circunferencias de centros O1 e O2, com o mesmo raio, que se intersectamem A e B. Seja P um ponto sobre o arco AB de C2 que esta dentro de C1. A reta APintersecta C1 em C, a reta CB intersecta C2 em D e a bissetriz de 6 CAD intersecta C1 emE e C2 em L. Seja F o simetrico do ponto D em relaao ao ponto medio de PE. Demonstrarque existe um ponto X que satisfaz 6 XFL = 6 XDC = 30◦ e CX = O1O2.

Cada problema vale 7 pontos.Tempo maximo da prova: 4 horas e meia.

24 Olimpiada Iberoamericana de Matematicas, Mexico.

Page 49: 24 Olimpiada Iberoamericana de Matemáticas

Problemas y Soluciones de la 24 OIM 45

Segundo dia

Problema 4Seja ABC um triangulo com AB 6= AC. Sejam I o incentro de ABC e P o outro ponto deinterseao da bissetriz externa do angulo A com o circuncırculo de ABC. A reta PI intersectao circuncırculo de ABC no ponto J . Demonstrar que os circuncırculos dos triangulos JIBe JIC sao tangentes as retas IC e IB, respectivamente.

Problema 5A sequencia an esta definida por

a1 = 1, a2k = 1 + ak e a2k+1 =1

a2k

, para todo inteiro k ≥ 1.

Demonstrar que todo numero racional positivo aparece exatamente uma vez nesta sequencia.

Problema 6Ao redor de uma circunferencia marcam-se 6000 pontos, cada um dos quais se pinta comuma de 10 cores dadas, de modo que entre quaisquer 100 pontos consecutivos sempre figuramas 10 cores. Achar o menor inteiro k com a seguinte propriedade: para toda coloraao destetipo existem k pontos consecutivos nos quais se encontram as 10 cores.

Cada problema vale 7 pontos.Tempo maximo da prova: 4 horas e meia.

24 Olimpiada Iberoamericana de Matematicas, Mexico.

Page 50: 24 Olimpiada Iberoamericana de Matemáticas

Actas de las reuniones de jurado de la24 OIM

Primer ActaEl Jurado de la 24 Olimpiada Iberoamericana de Matematicas se reune hoy Sabado 19

de Septiembre de 2009 a las 16:00 con el fin de llevar a cabo la primer reunion para laorganizacion del evento.

A continuacion se enlistan los asistentes a esta reunion.Como miembros del Jurado representantes del paıs organizador asisten:

• Radmila Bulajich Manfrino, Presidenta del Jurado

• Jose Antonio Gomez Ortega, Vicepresidente del Jurado

• Leonardo Ignacio Martınez Sandoval, Secretario del Jurado

Como Jefe de Delegacion de las delegaciones de los paıses participantes asisten:

• Patricia Fauring, Jefe de la Delegacion de Argentina

• Gustavo Michel Garcıa, Jefe de la Delegacion de Bolivia

• Onofre Campor Da Silva Farias, Jefe de la Delegacion de Brasil

• Juan Ignacio Restrepo, Jefe de la Delegacion de Colombia

• Jeremıas Ramırez Jimenez, Jefe de la Delegacion de Costa Rica

• Luis Rodrıguez Ojeda, Jefe de la Delegacion de Ecuador

• Eder Alexander Jacobo Arevalo, Jefe de la Delegacion de el Salvador

• William Roberto Gutierrez Herrera, Jefe de la Delegacion de Guatemala

• Luis Armando Ramos Palacios, Jefe de la Delegacion de Honduras

• Alejandro Bravo Mojica, Jefe de la Delegacion de Mexico

464646

Page 51: 24 Olimpiada Iberoamericana de Matemáticas

Actas de las reuniones de jurado de la 24 OIM 47

• Carlos Jose Walsh Mendoza, Jefe de la Delegacion de Nicaragua

• Gabriela Marıa Ratti Bittinger, Jefe de la Delegacion de Paraguay

• Jorge Joel Tipe Villanueva, Jefe de la Delegacion de Peru

• Joel Pedro de Oliveira Moreira, Jefe de la Delegacion de Portugal

• Luis Caseres, Jefe de la Delegacion de Puerto Rico

• Leonardo Lois Da Silva, Jefe de la Delegacion de Uruguay

• Henry Martınez Leon, Jefe de la Delegacion de Venezuela

Sin derecho a voto tambien participan en la sesion:

• Gabriela Gomez Pasquali, Observadora de Paraguay

• Juan Romero, Observador de Puerto Rico

• Rogelio Valdez Delgado, Jefe de Coordinaciones

• Florian Luca, Lıder del Comite de Seleccion de Problemas

Se llevaron a cabo las siguientes actividades.

• Presentacion del Jurado por parte de Jose Antonio Gomez Ortega.

• Presentacion de los Jefes de Delegacion por nombre y paıs.

• Se pone a votacion la participacion en la reunion de Florian Luca y Rogelio ValdezDelgado. Se vota a favor.

• Se pone a votacion la eliminacion del problema C3 por ser similar a un problema de laOlimpiada del Cono Sur de 1998. Se vota a favor.

• Se pone a votacion la eliminacion del problema N2 por ser similar a un problema dela Olimpiada Rioplatense de 2001. Se vota a favor.

• Se menciona que la tecnica utilizada en la segunda solucion del problema A1 es unatecnica utilizada frecuentemente en la resolucion de problemas. Se menciona tambienque esta tecnica aparece como consejo en libros. Los Jefes de Delegacion se dan porenterados.

• Se pone a votacion la eliminacion del problema A1 por las ideas mencionadas en elpunto anterior. Se vota a favor.

24 Olimpiada Iberoamericana de Matematicas, Mexico.

Page 52: 24 Olimpiada Iberoamericana de Matemáticas

48 Actas de las reuniones de jurado de la 24 OIM

• Se dan a conocer soluciones nuevas para los problemas A4, C6, G5 y G6. Se entregancopias de estas soluciones a los participantes en la reunion.

• Tiempo de trabajo para la solucion de los problemas.

• Los Delegados de cada paıs entregaron sus opiniones acerca de los problemas deposi-tando en una urna la Encuesta sobre la dificultad y pertinencia de los problemas.

• Se hicieron observaciones para los problemas de cada area (algebra, combinatoria,geometrıa, teorıa de numeros) hechas por los miembros que trabajaron en dicha area.Se hacen algunas observaciones generales.

• Se pone a votacion la eliminacion del problema G1 por ser similar a un problema dela Olimpiada de Sudafrica 2006. Se vota a favor.

• Se pone a votacion la eliminacion del problema C4 por tener ideas usadas frecuente-mente, y ser la version tridimensional de problemas conocidos que llenan un tablerocon 1 y -1 y piden posibles de sumas de configuraciones. Se vota a favor.

• Siendo las 19:17 se da por concluida la primer reunion del Jurado de la 24 OlimpiadaIberoamericana de Matematicas.

Segunda ActaEl Jurado de la 24 Olimpiada Iberoamericana de Matematicas se reune hoy Domingo

20 de Septiembre de 2009 a las 9:00 con el fin de llevar a cabo la segunda reunion para laorganizacion del evento.

Han asistido los miembros que asistieron la sesion pasada. Ademas, se unen a la reunion:

• Salvador Villegas Barranco, Jefe de la Delegacion de Espana

• Pedro Marrone Gaudiano, Jefe de la Delegacion de Panama

Se realizaron las siguientes actividades:

• Presentacion de los Jefes de Delegacion de Espana y Panama

• Resumen de los comentarios y actividades de la reunion anterior

• Se presentaron los resultados de la Encuestas sobre la dificultad y pertinencia de losproblemas

• Se propusieron parejas para problemas 1 y 4 del examen. Se vota a favor de la parejaC1-G3

• Se propone un grupo de trabajo para analizar mas a profundidad la dificultad de losproblemas propuestos como candidatos a 3 y 6

24 Olimpiada Iberoamericana de Matematicas, Mexico.

Page 53: 24 Olimpiada Iberoamericana de Matemáticas

Actas de las reuniones de jurado de la 24 OIM 49

• Tiempo de trabajo para profundizar en los problemas de dificultad media y difıcil

• El grupo de trabajo de problemas candidatos a 3 y 6 presenta sus resultados

• Se propone como pareja de problemas 3 y 6 a los problemas G8 y C5. Se vota a favor

• Se vota por las parejas de problemas medios. Se elije la pareja A5, N4

• Se vota por la eleccion del orden de los problemas propuestos para el examen, se elijeel siguiente orden:

– C1

– A5

– G8

– G3

– N4

– C5

• Se inicia trabajo para la revision y redaccion final de los enunciados de los problemas

• Siendo las 13:00 se da por concluida la segunda reunion del Jurado de la 24 OlimpiadaIberoamericana de Matematicas

Tercer ActaEl Jurado de la 24 Olimpiada Iberoamericana de Matematicas se reune hoy Domingo

20 de Septiembre de 2009 a las 16:00 con el fin de llevar a cabo la tercer reunion para laorganizacion del evento.

Han asistido los miembros que asistieron la sesion pasada.Se realizaron las siguientes actividades:

• Presentacion del trabajo realizado en la redaccion de los problemas

• Se realizan algunas modificaciones adicionales a los enunciados de los problemas

• Se vota a favor de las redacciones de los problemas del examen

• Se procede a un receso de 30 minutos

• Se realiza la traduccion del examen a idioma portugues a cargo de los jefes de delegacionde Portugal y Brasil

• Se presenta la version portuguesa frente al Jurado para su aprobacion. El Juradoaprueba la version portuguesa del examen

24 Olimpiada Iberoamericana de Matematicas, Mexico.

Page 54: 24 Olimpiada Iberoamericana de Matemáticas

50 Actas de las reuniones de jurado de la 24 OIM

• Siendo las 20:00 se da por concluida la tercer reunion del Jurado de la 24 OlimpiadaIberoamericana de Matematicas

Cuarta ActaEl Jurado de la 24 Olimpiada Iberoamericana de Matematicas se reune hoy Lunes 21 de

Septiembre de 2009 a las 9:00 con el fin de llevar a cabo la cuarta reunion para la organizaciondel evento.

Han asistido los miembros que asistieron la sesion pasada.Ademas se ha incorporado a la reunion

• Hernan Burgos Vega, Jefe de la Delegacion de Chile

Han participado tambien en la reunion los Coordinadores de los problemas para presentarel criterio de calificacion que les corresponde.

Se realizaron las siguientes actividades:

• Se entrega a los asistentes de la reunion la version preeliminar del examen.

• Se aprueban las versiones escritas del examen tanto en espanol como en portugues.

• Gabriela Gomez Pasquali, Observadora de Paraguay, pide un tiempo para tratar eltema de la formacion del Comite Ejecutivo Asesor. Se concede el tiempo.

• Dentro del tiempo concedido, Bolivia, Costa Rica y Panama muestran interes en or-ganizar las futuras ediciones de la Olimpiada Iberoamericana de Matematicas.

• Se acuerda que el Comite Ejecutivo Actual se reunira la noche de hoy para investi-gar acerca de antecedentes en la peticion de organizacion del evento previo y de ladeterminacion de los paıses organizadores para las ediciones 2011 y 2012 del Evento.

• Se acuerda que la formacion del Comite Ejecutivo Asesor se realizara una vez elegidolos paıses mencionados en el punto anterior y durante una de las reuniones posterioresdel Jurado

• Se procede a la presentacion de los Coordinadores del Problema 1:

– Efren Perez Terrazas

– Patricio Alva Tlacaelel

– Ana Rechtman

• Presentacion de la propuesta de criterios para la calificacion del Problema 1

• Se aprueba, bajo ciertas correcciones, el criterio para el Problema 1

24 Olimpiada Iberoamericana de Matematicas, Mexico.

Page 55: 24 Olimpiada Iberoamericana de Matemáticas

Actas de las reuniones de jurado de la 24 OIM 51

• Se procede a la presentacion de los Coordinadores del Problema 3:

– Hugo Villanueva

– Jesus Rodrıguez Viorato

– Edgardo Roldan

• Presentacion de la propuesta de criterios para la calificacion del Problema 3

• Se aprueba, bajo ciertas correcciones, el criterio para el Problema 3

• Se procede a la presentacion de los Coordinadores del Problema 4:

– Julio Rodrıguez

– Eduardo Velasco

– Antonio Olivas

• Presentacion de la propuesta de criterios para la calificacion del Problema 4

• Se aprueba, bajo ciertas correcciones, el criterio para el Problema 4

• El Jefe de Delegacion de Peru presenta una situacion especial debido a que es posibleque tres de sus participantes no lleguen a tiempo para presentar la primer pruebadescansados. Se propone, bajo el antecedente de una situacion similar ocurrida conla Delegacion de Cuba en la ... Olimpiada Internacional de Matemticas, que se lesposponga un poco el tiempo de prueba bajo las siguientes condiciones:

– Los participantes podran comenzar a presentar la prueba durante el transcursodel 22 de septiembre.

– Los participantes deberan permanecer aislados, tanto de los otros participantes,de su tutor y de medios electronicos hasta antes de realizar la prueba.

– El Comite Organizador se encargara de proporcionar un lugar para que los par-ticipantes realizen el examen, ası como a un encargado de supervisar su prueba.Los participantes contarn con las cuatro horas y media disponibles.

– Los participantes deberan presentar el segundo dıa de la prueba con los partici-pantes de los demas paıses.

• Se aprueba la propuesta para la situacion especial de la Delegacion de Peru

• Se procede a un receso de media hora

• Se procede a la presentacion de los Coordinadores del Problema 2:

– Humberto Montalvan

24 Olimpiada Iberoamericana de Matematicas, Mexico.

Page 56: 24 Olimpiada Iberoamericana de Matemáticas

52 Actas de las reuniones de jurado de la 24 OIM

– Manuel Guevara

– Enrique Trevino

• Presentacion de la propuesta de criterios para la calificacion del Problema 2

• Se aprueba el criterio para el Problema 2

• Se procede a la presentacion de los Coordinadores del Problema 5:

– Marıa Eugenia Guzman

– David Cossio

– Joshua Hernandez

• Presentacion de la propuesta de criterios para la calificacion del Problema 5

• Se aprueba, bajo ciertas correcciones, el criterio para el Problema 5

• Se procede a la presentacion de los Coordinadores del Problema 6:

– Bernardo Abrego

– Jacob Rubio

– Carlos Villalvazo

• Presentacion de la propuesta de criterios para la calificacion del Problema 6

• Se aprueba el criterio para el Problema 6

• Siendo las 13:00 se da por concluida la cuarta reunion del Jurado de la 24 OlimpiadaIberoamericana de Matematicas

Quinta ActaEl Jurado de la 24 Olimpiada Iberoamericana de Matematicas se reune hoy Martes 22 de

Septiembre de 2009 a las 9:00 con el fin de llevar a cabo la quinta reunion para la organizaciondel evento.

Han asistido los mismos miembros que a la reunion pasada.Se realizaron las siguientes actividades:

• Tiempo para la resolucion de las dudas de los participantes que surgieron el primer dıade la prueba. Se recibieron en total 41 preguntas

• Se abre la discusion para la decision de las sedes para las proximas ediciones de laOlimpiada Iberoamericana de Matematicas. Se presentan las siguientes consideracionespor parte de los paıses interesados

24 Olimpiada Iberoamericana de Matematicas, Mexico.

Page 57: 24 Olimpiada Iberoamericana de Matemáticas

Actas de las reuniones de jurado de la 24 OIM 53

– Panama: Plantea que es posible organizar el Evento en 2012, pero por razoneselectorales en su paıs, considera difıcil postergala un poco mas.

– Costa Rica: Comenta que Costa Rica presento su oferta hace dos anos y la reiterohace uno. Considera que postergarla mas es muy difıcil.

– Bolivia: El Jefe de Delegacion de Bolivia plantea que piensa retirarse academica-mente en un par de anos. Comenta que de organizarse el Evento del Bolivia en elano 2012, todavıa podrıa tener los contactos adecuados para organizarla. Despuesde ese periodo, sigue abierta la oferta, pero comenta que posiblemente ya no seael quien la organize y que el compromiso no es tan concreto, y se tendrıa quedecidir durante la Olimpiada Iberoamericana en Paraguay en 2010.

– Puerto Rico: Abre la oferta de organizar la Olimpiada para el ano 2013. Consideraque es posible postergarlo un poco mas, siempre y cuando se le avise con dos anosde anticipacion.

• El Jurado se da por enterado de estas propuestas

• Panama consultara si puede postergar la organizacion de la Olimpiada un ano. Infor-mara su resolucion al Comite Ejecutivo asesor dentro de los siguientes tres meses

• Se reitera el compromiso con mantener en secreto la lista corta de los problemas hastaSeptiembre de 2010

• Se acuerda entregar una lista corta extra a cada uno de los Jefes de Delegacion. CadaJefe de Delegacion decidira si desea entregar la lista corta al tutor de su Delegacion ono

• Siendo las 10:20 se da por concluida la quinta reunion del Jurado de la 24 OlimpiadaIberoamericana de Matematicas

Sexta ActaEl Jurado de la 24 Olimpiada Iberoamericana de Matematicas se reune hoy Miercoles

23 de Septiembre de 2009 a las 9:00 con el fin de llevar a cabo la sexta reunion para laorganizacion del evento.

Han asistido los mismos miembros que a la reunion pasada.Se realizaron las siguientes actividades:

• Tiempo para la resolucion de las dudas de los participantes que surgieron el segundodıa de la prueba. Se recibieron un total de 21 preguntas.

• Siendo las 9:43 se da por concluida la tercer reunion del Jurado de la 24 OlimpiadaIberoamericana de Matematicas.

24 Olimpiada Iberoamericana de Matematicas, Mexico.

Page 58: 24 Olimpiada Iberoamericana de Matemáticas

54 Actas de las reuniones de jurado de la 24 OIM

Septima ActaEl Jurado de la 24 Olimpiada Iberoamericana de Matematicas se reune hoy Martes 22

de Septiembre de 2009 a las 23:00 con el fin de llevar a cabo la septima reunion para laorganizacion del evento.

Han asistido los mismos miembros que a la reunion pasada. Se ha unido a la reunion losTutores de los paıses y algunos observadores adicionales sin derecho a voto.

Se realizaron las siguientes actividades:

• Bernardo Abrego, invitado, dio a conocer a los Delegados de los paıses y al Juradoque los paıses de latinoamerica tambien pueden participar en la AMC, concurso dematematicas norteamericano

• Se presentaron los resultados de los puntajes por examen sin los nombres de los par-ticipantes. Con base en estos resultados se voto por los siguientes cortes:

– Oro: 38 puntos

– Plata: 32 puntos

– Bronce: 24 puntos

• Espana reporta un error de captura en los datos de uno de sus concursantes. Se corrigeeste error

• Con base en el error corregido, los cortes para medallas se cambian a los siguientes:

– Oro: 38 puntos

– Plata: 31puntos

– Bronce: 24 puntos

• Jose Antonio Gomez Ortrega hace el aviso de que al publicarse el examen en M athlinks,portal electronico de problemas de matematicas, hubo usuarios de ese sitio que infor-maron que el problema 5 del examen ya habıa sido publicado previamente. Los Dele-gados se dan por eterados y reiteran el comrpomiso de verificar la originalidad de losproblemas

• Radmila Bulajich informa acerca de un estudiante que regreso a su casa sin avisar a laorganizacion del evento. Informa que este tipo de actitudes afectan a la organizaciondel examen pues inquieta a los guıas responsables del estudiante y generan cargosadicionales que pudieron ser evitados. Hace un llamado a que se de aviso previo paracontemplar las acciones que puedan mejorar al evento. Los Delegados se dan porenterados. El Delegado de Honduras asume la responsabilidad por lo ocurrido

• Fernando Campos Garcıa propone abrir a discusion la regla de que un estudiante puedaparticipar solo dos veces en la Olimpiada Iberoamericana de Matematicas. Se proponeque se ponga a consideracion por parte del Comite Ejecutivo Asesor. Se vota en contra

24 Olimpiada Iberoamericana de Matematicas, Mexico.

Page 59: 24 Olimpiada Iberoamericana de Matemáticas

Actas de las reuniones de jurado de la 24 OIM 55

• Entrega de las invitaciones para la 24 Olimpiada Iberoamericana de Matematicas porparte de la Gabriela Gomez Pascuali en nombre de Paraguay.

• La Delegada de Argentina felicita al Comite Organidor por su labor en la organizaciondel evento

• Siendo las 11:40 se da por concluida la Septima reunion del Jurado de la 24 OlimpiadaIberoamericana de Matematicas

Constitucion del Comite Ejecutivo Asesor

• Un representante del proximo organizador - Paraguay (presidente) - Gabriela GomezPascualli

• Un representante del actual organizador - Mexico (secretario) - Radmila Bulajich Man-frino

• Un representante del anterior organizador - Brasil (vocal) - Carlos Yuzo Shine

• Un representante de la edicion 2011 - Costa Rica(vocal) - Mario Marın Sanchez

• Un representante del jurado - Guatemala (vocal) - Mayra Castillo

Comite Ejecutivo Asesor Actual

• Un representante del proximo organizador - Mexico (presidente) - Radmila BulajichManfrino

• Un representante del actual organizador - Brasil (secretario) - Luzinalva Miranda deAmorin

• Un representante del anterior organizador - Portugal (vocal) - Joana Teles Correira

• Un representante de la edicion 2011 - Paraguay (vocal) - Gabriela Gomez Pascualli

• Un representante del jurado - Espana (vocal) - Juan Manuel Conde

24 Olimpiada Iberoamericana de Matematicas, Mexico.

Page 60: 24 Olimpiada Iberoamericana de Matemáticas

Premios de la 24 OIM

Distribucion de Premios

Premio Puntaje Num Sum Rango Sum(%)42 0 0 1 0.0%41 1 1 1 1.2%

Oro 40 0 1 2 1.2%39 1 2 2 2.5%38 5 7 3 8.6%37 1 8 8 9.9%36 3 11 9 13.6%35 3 14 12 17.3%

Plata 34 1 15 15 18.5%33 3 18 16 22.2%32 1 19 19 23.5%31 2 21 20 25.9%30 3 24 22 29.6%29 4 28 25 34.6%28 4 32 29 39.5%

Bronce 27 1 33 33 40.7%26 1 34 34 42.0%25 2 36 35 44.4%24 5 41 37 50.6%

565656

Page 61: 24 Olimpiada Iberoamericana de Matemáticas

Premios de la 24 OIM 57

23 2 43 42 53.1%22 1 44 44 54.3%21 2 46 45 56.8%20 2 48 47 59.3%19 2 50 49 61.7%18 2 52 51 64.2%17 2 54 53 66.7%16 3 57 55 70.4%15 2 59 58 72.8%14 3 62 60 76.5%13 5 67 63 82.7%12 2 69 68 85.2%11 0 69 70 85.2%10 3 72 70 88.9%9 3 75 73 92.6%8 0 75 76 92.6%7 2 77 76 95.1%6 1 78 78 96.3%5 0 78 79 96.3%4 0 78 79 96.3%3 1 79 79 97.5%2 1 80 80 98.8%1 1 81 81 100.0%0 0 81 82 100.0%

24 Olimpiada Iberoamericana de Matematicas, Mexico.

Page 62: 24 Olimpiada Iberoamericana de Matemáticas

Puntaje por participante

ArgentinaConcursante P1 P2 P3 P4 P5 P6 Final Medalla

ARG1 7 7 6 7 7 3 37 PlataARG2 7 5 4 6 6 1 29 BronceARG3 7 7 0 7 7 4 32 PlataARG4 4 6 2 7 7 4 30 BronceTOTAL 25 25 12 27 27 12 128

BoliviaConcursante P1 P2 P3 P4 P5 P6 Final Medalla

BOL2 5 1 0 0 0 0 6BOL3 0 1 0 0 0 0 1BOL4 6 1 1 0 1 0 9

TOTAL 11 3 1 0 1 0 16

BrasilConcursante P1 P2 P3 P4 P5 P6 Final Medalla

BRA1 7 7 5 7 5 2 33 PlataBRA2 7 7 2 7 7 3 33 PlataBRA3 7 7 7 7 7 3 38 OroBRA4 7 7 6 7 7 7 41 Oro

TOTAL 28 28 20 28 26 15 145

ChileConcursante P1 P2 P3 P4 P5 P6 Final Medalla

CHL1 6 7 2 7 7 0 29 BronceCHL2 7 1 0 7 0 0 15 M. HonorıficaCHL3 7 2 0 7 2 3 21 M. HonorıficaCHL4 6 1 0 7 1 1 16 M. Honorıfica

TOTAL 26 11 2 28 10 4 81

585858

Page 63: 24 Olimpiada Iberoamericana de Matemáticas

Puntaje por participante 59

ColombiaConcursante P1 P2 P3 P4 P5 P6 Final Medalla

COL1 7 7 4 7 7 7 39 OroCOL2 7 2 0 7 7 1 24 BronceCOL3 7 6 7 7 7 1 35 PlataCOL4 7 2 0 7 7 0 23 M. Honorıfica

TOTAL 28 17 11 28 28 9 121

Costa RicaConcursante P1 P2 P3 P4 P5 P6 Final Medalla

CRI1 7 1 0 3 3 0 14 M. HonorıficaCRI2 7 1 0 3 7 1 19 M. HonorıficaCRI3 7 7 0 7 5 1 27 BronceCRI4 3 3 0 7 0 0 13 M. Honorıfica

TOTAL 24 12 0 20 15 2 73

CubaConcursante P1 P2 P3 P4 P5 P6 Final Medalla

CUB1 7 7 7 7 7 3 38 OroCUB2 6 7 1 7 7 1 29 BronceCUB3 7 7 1 7 3 0 25 BronceCUB4 7 7 1 7 1 1 24 Bronce

TOTAL 27 28 10 28 18 5 116

EcuadorConcursante P1 P2 P3 P4 P5 P6 Final Medalla

ECU1 6 7 0 0 6 0 19 M. HonorıficaECU2 6 0 1 7 0 0 14 M. HonorıficaECU3 7 4 0 7 5 1 24 BronceECU4 0 1 0 7 7 1 16 M. Honorıfica

TOTAL 19 12 1 21 18 2 73

El SalvadorConcursante P1 P2 P3 P4 P5 P6 Final Medalla

SLV1 7 7 0 7 4 0 25 BronceSLV2 6 6 0 7 7 0 26 BronceSLV3 6 7 2 7 1 0 23 M. HonorıficaSLV4 3 1 0 7 7 0 18 M. Honorıfica

TOTAL 22 21 2 28 19 0 92

24 Olimpiada Iberoamericana de Matematicas, Mexico.

Page 64: 24 Olimpiada Iberoamericana de Matemáticas

60 Puntaje por participante

EspanaConcursante P1 P2 P3 P4 P5 P6 Final Medalla

ESP1 4 7 7 7 6 5 36 PlataESP2 7 7 2 7 7 1 31 PlataESP3 6 7 1 7 7 3 31 PlataESP4 7 7 2 1 7 4 28 Bronce

TOTAL 24 28 12 22 27 13 126

GuatemalaConcursante P1 P2 P3 P4 P5 P6 Final Medalla

GTM1 7 7 1 5 7 1 28 BronceGTM2 7 1 0 0 2 0 10 M. HonorıficaGTM3 3 0 0 0 0 0 3GTM4 7 1 0 0 1 3 12 M. HonorıficaTOTAL 24 9 1 5 10 4 53

HondurasConcursante P1 P2 P3 P4 P5 P6 Final Medalla

HND1 6 6 1 7 7 1 28 BronceHND2 5 1 1 7 2 1 17 M. HonorıficaHND3 5 1 0 7 0 0 13 M. HonorıficaHND4 5 0 1 7 7 0 20 M. HonorıficaTOTAL 21 8 3 28 16 2 78

MexicoConcursante P1 P2 P3 P4 P5 P6 Final Medalla

MEX1 7 7 2 7 7 3 33 PlataMEX2 7 7 2 7 7 5 35 PlataMEX3 7 7 3 7 7 7 38 OroMEX4 7 1 1 7 2 0 18 M. HonorıficaTOTAL 28 22 8 28 23 15 124

NicaraguaConcursante P1 P2 P3 P4 P5 P6 Final Medalla

NIC1 6 0 6 7 2 0 21 M. HonorıficaNIC2 0 1 1 7 0 0 9 M. HonorıficaNIC3 0 1 0 1 0 0 2NIC4 4 1 0 7 1 0 13 M. Honorıfica

TOTAL 10 3 7 22 3 0 45

24 Olimpiada Iberoamericana de Matematicas, Mexico.

Page 65: 24 Olimpiada Iberoamericana de Matemáticas

Puntaje por participante 61

PanamaConcursante P1 P2 P3 P4 P5 P6 Final Medalla

PAN1 7 7 1 7 7 1 30 BroncePAN2 7 5 0 7 7 3 29 BroncePAN3 7 6 1 1 5 0 20 M. Honorıfica

TOTAL 21 18 2 15 19 4 79

ParaguayConcursante P1 P2 P3 P4 P5 P6 Final Medalla

PRY1 1 1 1 7 0 0 10 M. HonorıficaPRY2 4 1 0 3 7 0 15 M. HonorıficaPRY3 3 2 0 2 0 0 7PRY4 1 0 0 7 2 0 10 M. Honorıfica

TOTAL 9 4 1 19 9 0 42

PeruConcursante P1 P2 P3 P4 P5 P6 Final Medalla

PER1 7 7 7 7 5 1 34 PlataPER2 6 7 7 7 7 2 36 PlataPER3 7 7 7 7 7 3 38 OroPER4 7 7 7 7 7 3 38 Oro

TOTAL 27 28 28 28 26 9 146

PortugalConcursante P1 P2 P3 P4 P5 P6 Final Medalla

PRT1 7 2 0 1 2 1 13 M. HonorıficaPRT2 7 4 1 7 5 0 24 BroncePRT3 7 7 6 7 7 1 35 PlataPRT4 7 7 3 7 7 5 36 Plata

TOTAL 28 20 10 22 21 7 108

Puerto RicoConcursante P1 P2 P3 P4 P5 P6 Final Medalla

PRI1 7 1 0 7 1 0 16 M. HonorıficaPRI2 6 7 3 7 7 0 30 BroncePRI3 6 1 0 0 1 1 9PRI4 7 6 0 3 1 0 17 M. Honorıfica

TOTAL 26 15 3 17 10 1 72

24 Olimpiada Iberoamericana de Matematicas, Mexico.

Page 66: 24 Olimpiada Iberoamericana de Matemáticas

62 Puntaje por participante

UruguayConcursante P1 P2 P3 P4 P5 P6 Final Medalla

URY1 7 0 4 1 0 1 13 M. HonorıficaURY2 7 7 0 7 7 0 28 BronceURY3 7 6 0 7 0 4 24 BronceURY4 3 1 1 7 0 0 12 M. Honorıfica

TOTAL 24 14 5 22 7 5 77

VenezuelaConcursante P1 P2 P3 P4 P5 P6 Final Medalla

VEN1 4 5 0 7 5 1 22 M. HonorıficaVEN2 7 0 0 7 0 0 14 M. HonorıficaVEN3 6 0 0 1 0 0 7

TOTAL 17 5 0 15 5 1 43

24 Olimpiada Iberoamericana de Matematicas, Mexico.

Page 67: 24 Olimpiada Iberoamericana de Matemáticas

Distribucion por problema y puntaje

Problema 1

0

20

40

60

80

4

0

2

1

0

2

5

3

5

4

4

5

15

6

46

7

Calificacion Promedio = 5.79

Num

ero

Problema 2

0

20

40

60

80

8

0

22

1

5

2

1

3

2

4

3

5

7

6

33

7

Calificacion Promedio = 4.08

Num

ero

Problema 3

0

20

40

60

80

37

0

17

1

8

2

3

3

3

4

1

5

4

6

8

7

Calificacion Promedio = 1.71

Num

ero

Problema 4

0

20

40

60

80

8

0

6

1

1

2

4

3

0

4

1

5

1

6

60

7

Calificacion Promedio = 5.56

Num

ero

Problema 5

0

20

40

60

80

16

0

9

1

7

2

2

3

1

4

7

5

3

6

36

7

Calificacion Promedio = 4.17

Num

ero

Problema 6

0

20

40

60

80

37

0

21

1

2

2

11

3

4

4

3

5

0

6

3

7

Calificacion Promedio = 1.35

Num

ero

636363

Page 68: 24 Olimpiada Iberoamericana de Matemáticas

64 Distribucion por problema y puntaje

0

1

2

3

4

5

6

0

0

1

1

1

2

1

3

0

4

0

5

1

6

2

7

0

8

3

9

3

10

0

11

2

12

5

13

3

14

2

15

316

2

17

2

18

2

19

2

20

2

21

1

222

23

5

24

2

25

1

26

1

27

4

28

4

29

3

30

2

31

1

32

3

33

1

34

3

35

3

36

1

37

5

38

1

39

0

40

1

41

0

42

Puntaje

Num

ero

de

Per

sonas

24 Olimpiada Iberoamericana de Matematicas, Mexico.

Page 69: 24 Olimpiada Iberoamericana de Matemáticas

Promedio por problema

Puntaje P1 P2 P3 P4 P5 P67 46 33 8 60 36 36 15 7 4 1 3 05 4 3 1 1 7 34 5 2 3 0 1 43 5 1 3 4 2 112 0 5 8 1 7 21 2 22 17 6 9 210 4 8 37 8 16 37

Promedio 5.79 4.08 1.71 5.56 4.17 1.35Dificultad 6 3 2 5 4 1

656565

Page 70: 24 Olimpiada Iberoamericana de Matemáticas

Medallas

Medallistas de Oro

Brasil Finder Renan HenriqueBrasil Secco Torres da Silva MatheusColombia Olarte Parra Jorge AlbertoCuba Gil Pons ReynaldoMexico Lopez Buenfil Manuel GuillermoPeru Guerra Rios PercyPeru Ramos Castillo Ricardo Jesus

Medallistas de Plata

Argentina Maurizio Miguel Sebastian

Argentina Stefanich German Ezequiel

Brasil de Sa Oliveira Sales Marcelo Tadeu

Brasil Lopes Pedroso Marco Antonio

Colombia Del Castillo Munoz Nicolas Eduardo

Espana Bello Burguet Glenier Lazaro

Espana Geffner Fuenmayor Ivan

Espana Herradon Cueto Moises

Mexico Gallegos Banos Erik Alejandro

Mexico Perales Anaya Daniel

Peru Mejıa Cordero Julian Alonso

Peru Velez Salamanca Amilcar Enrique

Portugal Landeira da Silva Miranda Jorge Ricardo

Portugal Passos de Sousa Vieira Pedro Manuel

666666

Page 71: 24 Olimpiada Iberoamericana de Matemáticas

Medallas 67

Medallistas de Bronce

Argentina Dodyk Juan Andres

Argentina Sadofschi Costa Ivan

Chile Velozo Anibal

Colombia Arcila Pardo David Mauricio

Costa Rica Vargas Ramırez Julio Andres

Cuba Estrada Hernandez Jorge

Cuba Moraguez Pinol Jose

Cuba Otero Baguer Daniel

Ecuador Bravo Barahona Gabriel Isaac

El Salvador Alberti Arroyo Hector Enmanuel

El Salvador Ayala Menjivar Julio Cesar

Espana Lamaison Vidarte Ander

Guatemala Vargas De Leon Alejandro Jose

Honduras Madrid Jose Ramon

Panama Fan Huang Antonio

Panama Hernandez Anguizola Edwin

Portugal Magalhaes dos Santos Joao Miguel

Puerto Rico Arzeno Soltero George

Uruguay Garcia Juan Pablo

Uruguay Rodrıguez Brena Ismael Valentın

Mencion Honorıfica

Chile Chanique Andrea

Chile Garcia Felipe

Chile Toro Valentina

Colombia Dıaz Carrillo Carlos Andres

Costa Rica Mora Saenz German Jose

Costa Rica Rodrıguez Roman Ruben

Costa Rica Valverde Lizano Jonathan

Ecuador Abad Coronel Christian Fernando

Ecuador Figueroa Mendoza Jeniffer Staphania

Ecuador Ordonez Mera Miguel Angel

24 Olimpiada Iberoamericana de Matematicas, Mexico.

Page 72: 24 Olimpiada Iberoamericana de Matemáticas

68 Medallas

El Salvador Bermudez Huezo Jaime Antonio

El Salvador Hernandez Cruz Nahomy Jhopselyn

Guatemala Ligorrıa Taracena Jose Alberto

Guatemala Mazariegos Camas Fernando Jose

Honduras Bermudez Nestor Alejandro

Honduras Madrid Padilla Oscar Hernan

Honduras Manzanarez Sergio David

Mexico Bibiano Velasco Cesar Daniel

Nicaragua Casco Arevalo Rufo Holvin

Nicaragua Castro Soza Hans Mijail

Nicaragua Duarte Vallejos Silvio Francisco

Panama Wong Yau Ricardo

Paraguay Benıtez Maidana Rodrigo Daniel

Paraguay Elizeche Armoa Edgar Federico

Paraguay Quinonez Vazquez Osmar Mateo

Portugal Oliveira Toulson Frederico

Puerto Rico Arun Aravind

Puerto Rico Wagner Rodrıguez Alan

Uruguay Cotto Martin

Uruguay Escuder Rebori Matıas

Venezuela Acevedo Castillo Carmela

Venezuela Marcano Sandoval Mauricio

24 Olimpiada Iberoamericana de Matematicas, Mexico.

Page 73: 24 Olimpiada Iberoamericana de Matemáticas

Medallas por paıs

Paıs O P B MH # C TotArgentina 0 2 2 0 4 128Bolivia 0 0 0 0 3 16Brasil 2 2 0 0 4 145Chile 0 0 1 3 4 81Colombia 1 1 1 1 4 121Costa Rica 0 0 1 3 4 73Cuba 1 0 3 0 4 116Ecuador 0 0 1 3 4 73El Salvador 0 0 2 2 4 92Espana 0 3 1 0 4 126Guatemala 0 0 1 2 4 53Honduras 0 0 1 3 4 78Mexico 1 2 0 1 4 124Nicaragua 0 0 0 3 4 45Panama 0 0 2 1 3 79Paraguay 0 0 0 3 4 42Peru 2 2 0 0 4 146Portugal 0 2 1 1 4 108Puerto Rico 0 0 1 2 4 72Uruguay 0 0 2 2 4 77Venezuela 0 0 0 2 3 43

696969

Page 74: 24 Olimpiada Iberoamericana de Matemáticas

Resultado Copa Puerto Rico

2007 2008 2009

Puntuacion Puntuacion Puntuacion P Q S

ARG 108 124 128 164.23 137.42 -43.23

BOL 3 27 16 28.31 17.17 -13.96

BRA 114 155 145 190.42 155.67 -53.79

CHL 52 78 81 92.02 86.96 -14.26

COL 73 94 121 118.21 129.90 -0.13

CRI 35 77 73 79.28 78.37 -8.83

CUB 48 130 116 126 124.53 -14.06

SLV 52 95 92 104.06 98.77 -15.69

ECU 44 79 73 87.07 78.37 -17.4

ESP 60 121 126 128.12 135.27 -5.66

GTM 27 27 53 38.22 56.9 14.85

HND 32 46 78 55.21 83.74 23

MEX 90 115 124 145.11 133.12 -26.5

PAN 36 30 79 62.29 84.81 16.29

PRY 42 30 42 50.96 45.09 -10.97

PER 91 150 146 170.6 156.74 -30.91

PRI 43 50 72 65.83 77.3 4.88

PRT 66 92 108 111.84 115.95 -7.08

URU 44 77 77 85.65 82.66 -11.55

VEN 28 52 43 64.72 46.16 -25.02

707070

Page 75: 24 Olimpiada Iberoamericana de Matemáticas

Ceremonia de clausura 24 OIM

CEREMONIA DE CLAUSURASabado 26 de septiembre de 2009, 17:00 horas

Museo Regional de Queretaro

Introduccion

Presentacion del Presıdium

Mensaje de Dr. Alejandro Lozano Guzman, Director General del Consejo de Ciencia y Tec-nologıa del Estado de Queretaro.

Mensaje de Dra. Radmila Bulajich Manfrino, Presidenta del Comite Organizador de la OIM.

Premiacion

Mensaje de Dr. Gilberto Herrera Ruiz, Director de la Facultad de Ingenierıa de la Univer-sidad Autonoma de Queretaro.

Proyeccion de Video de la 24 OIM

Presentacion de la Sede de la 25 OIM

Despedida

717171

Page 76: 24 Olimpiada Iberoamericana de Matemáticas

72 Ceremonia de clausura 24 OIM

IntroduccionBuenas tardes a todas las personas que nos acompanan hoy, en esta ceremonia de clausura

de la vigesima cuarta olimpiada iberoamericana de matematicas. Es un orgullo para todosnosotros llegar a la culminacion de este evento, que sin duda, con la cooperacion activa yresponsable de todos ustedes fue posible finalizar exitosamente.

Discurso clausura: Dra. Radmila Bulajich ManfrinoHonorables miembros del Presidium, concursantes y colegas.Participar en una Olimpiada Iberoamericana significa mucho mas que presentar dos

examenes. Por lo tanto, quiero dedicar mis primeras palabras a reconocer los muchos dıas,fines de semana y concienzudas horas de estudio que cada uno de los concursantes dedico aprepararse para esta ocasion.

Es patente el orgullo que sentimos al otorgar, a los jovenes aquı presentes, los premiosque han ganado gracias a su teson, inteligencia y entrega al estudio de las matematicas. Ellosproceden de los mas distintos escenarios, pero se parecen en una cosa: son todos duenos deun loable deseo de superacion.

Quiero que celebremos su trabajo, pues creo muy sinceramente que todos pueden estarsatisfechos. Incluso si muchos de entre ustedes no obtuvieron los resultados que esperaban,es importante que no olviden esta semana de convivencia, y que recuerden que haber llegadohasta aquı representa en sı un gran logro del que deben sentirse orgullosos.

Todos los aquı presentes sabemos que en esta semana de Olimpiada se ha mezclado elplacer con el trabajo, las risas con la reflexion y las matematicas con las buenas charlas.Sepan que muchas de las personas que conocieron en esta semana, seran probablemente suscompaneros durante toda la vida. Estos encuentros abren puertas hacia nuevos horizontes.

A pesar de que para la mayorıa de nosotros trabajar para la Olimpiada es una laboraltruista, seguimos participando en ella. Lo hacemos por una simple razon: el placer quenos produce, ano con ano, conocer a las nuevas generaciones. Verlos posteriormente crecer,y trabajar con entusiasmo por el futuro de nuestros paıses.

La 24 Olimpiada Iberoamericana de Matematicas ha llegado a su fin. En todas las etapasde nuestras vidas hay momentos agradables y desagradables, se sufren vicisitudes, peroesperamos ante todo que la experiencia durante estos dıas les haya resultado enriquecedora.

Quiero agradecer a todos aquellos que participan y organizan todas y cada una de lasetapas de la Olimpiada.

Deseo tambien extender nuestras felicitaciones a todos quienes a traves de su labor diariafomentan la cultura matematica en nuestro paıs y en iberoamerica.

Mi mas sincero reconocimiento a todas las Instituciones que hacen posible que las Olim-piadas continuen vivas.

Quisiera agradecer nuevamente a la Universidad Autonoma de Queretaro, a la Secretarıade Educacion Publica del Estado y a la federal, al Consejo de Ciencia y Tecnologıa deQueretaro y al Nacional, a la Universidad de Guanajuato, al Centro de Investigacion enMatematicas y a todas las instituciones que nos apoyaron. Agradezco de manera muy especial

24 Olimpiada Iberoamericana de Matematicas, Mexico.

Page 77: 24 Olimpiada Iberoamericana de Matemáticas

Ceremonia de clausura 24 OIM 73

a Carmen Sosa y su equipo. A los guıas, ya que ellos constituyen una parte fundamentalen el desarrollo de las actividades y a Nacho que ano con ano, olimpiada con olimpiada, harealizado una excelente labor como papa de todos los alumnos. Por ultimo a todo el comiteorganizador, al comite de seleccion de problemas y a los coordinadores por todo su trabajo.

Esperamos que el recuerdo de esta olimpiada les sea grato, que los acompane siempre yque los inspire para seguir haciendo de la reflexion matematica una de las riquezas de suvida diaria.

Una vez mas, muchas felicidades a todos los participantes.Gracias.

PremiacionDemos nuestra mas significativa enhorabuena a los jovenes que participaron en este en-

cuentro de creatividad, razonamiento y reto intelectual, donde el talento de cada paıs ibero-americano dio muestra de la mejor aptitud y actitud ante las matematicas. Hemos llegadoal momento de reconocer a quienes han sobresalido en la ejecucion de su prueba.

ClausuraUna vez mas, la Universidad Autonoma de Queretaro, cumple con su mision, como

Maxima Casa de Estudios del Estado. Con el apoyo recibido se manifiesta la importanciaque para esta institucion tienen estos eventos de alta envergadura como lo es esta olimpiada.Es ası como la UAQ reafirma su compromiso con el quehacer academico y cultural que re-quiere la sociedad.

DespedidaNuestra gratitud a las instituciones que nos apoyaron para que este evento fuera posible.

A las autoridades que con su presencia dan realce a esta ceremonia, muchas gracias por estaaquı. Agradecemos nuevamente la disposicion y entusiasmo puesto en todas las actividades delos coordinadores, guıas, lıderes y tutores. A los jovenes participantes, les pedimos que estano sea la ultima vez que intervengan en este movimiento que es la Olimpiada de Matematicas.

24 Olimpiada Iberoamericana de Matematicas, Mexico.